Immersion Cardiac

Ace your homework & exams now with Quizwiz!

36. Which of the following is a late symptom of polycythemia vera? ■ 1. Headache. ■ 2. Dizziness. ■ 3. Pruritus. ■ 4. Shortness of breath.

3. Pruritus is a late symptom that results from abnormal histamine metabolism. Headache and dizziness are early symptoms from engorged veins. Shortness of breath is an early symptom from congested mucous membranes and ineffective gas exchange.

The nurse has applied the patch electrodes of an automatic external defibrillator (AED) to the chest of a client who is pulseless. The defibrillator has interpreted the rhythm to be ventricular fibrillation. The nurse then: 1 Administers rescue breathing during the defibrillation 2 Performs cardiopulmonary resuscitation (CPR) for 1 minute before defibrillating 3 Charges the machine and immediately pushes the "discharge" buttons on the console 4 Orders any personnel away from the client, charges the machine, and defibrillates through the console

4 If the AED advises to defibrillate, the nurse or rescuer orders all persons away from the client, charges the machine, and pushes both of the "discharge" buttons on the console at the same time. The charge is delivered through the patch electrodes, and this method is known as "handsoff" defibrillation, which is safest for the rescuer. The sequence of charges is similar to that of conventional defibrillation. Option 1 is contraindicated for the safety of any rescuer. Performing CPR delays the defibrillation attempt.

clinic nurse provides home care instructions to an adolescent with iron deficiency anemia about the administration of oral iron preparations. The nurse tells the adolescent that it is best to take the iron with: 1 Cola 2 Soda 3 Water 4 Tomato juice

4 Iron should be administered with vitamin C-rich fluids, because vitamin C enhances the absorption of the iron preparation. Tomato juice has a high ascorbic acid (vitamin C) content, whereas water, soda, and cola do not contain vitamin C.

A nurse is assessing a blood pressure of an adult client with a manual sphygmomanometer. The nurse places the bell diaphragm of the stethoscope over the brachial artery and pumps the cuff up to 180 mm Hg. The valve is released to allow a drop of 2 mm Hg per second. At 162 mm Hg the nurse hears the first tapping sound. The sound becomes muffled at 148 mm Hg. The sound changes to a soft thumping at the 138 mm Hg. The sound fades to a muffled blowing sound at 128 mm Hg and is last heard at 94 mm Hg. There is silence at 92 mm Hg. The nurse should document the blood pressure as: 1. 138/92 mm Hg. 2. 148/94 mm Hg. 3. 162/92 mm Hg. 4. 162/94 mm Hg.

4 The systolic blood pressure is elicited at the pressure where the first clear tapping sound is heard. The diastolic blood pressure is elicited at the pressure where the last sound is heard

132. A client has the following arterial blood gas values: pH, 7.52; PaO2, 50 mm Hg; PaCO2, 28 mm Hg; HCO3 -, 24 mEq/L. The nurse determines that which of the following is a possible cause for these fi ndings? ■ 1. Chronic obstructive pulmonary disease (COPD). ■ 2. Diabetic ketoacidosis with Kussmaul's respirations. ■ 3. Myocardial infarction. ■ 4. Pulmonary embolus.

4. A PaCO2 of 28 mm Hg and PaO2 of 50 mm Hg are both abnormal; the PaO2 of 50 mm Hg signifi es acute respiratory failure. In evaluating possible causes for this disorder, the nurse should consider conditions that lead to hypoxia and hyperventilation, such as pulmonary embolus. COPD is typically associated with respiratory acidosis and elevated PaCO2. The client with diabetic ketoacidosis most often has metabolic acidosis. A myocardial infarction does not often cause an acid-base imbalance because the primary problem is cardiac in origin.

121. A client's chest tube is to be removed by the physician. Which of the following items should the nurse have ready to be placed directly over the wound when the chest tube is removed? ■ 1. Butterfl y dressing. ■ 2. Montgomery strap. ■ 3. Fine-mesh gauze dressing. ■ 4. Petroleum gauze dressing.

4. Immediately after chest tube removal, a petroleum gauze is placed over the wound and covered with a dry sterile dressing. This serves as an airtight seal to prevent air leakage or air movement in either direction. Bandages are not applied directly over wounds. Montgomery straps are used in place of adhesive tape when a dressing requires very frequent changes and the constant removal of adhesive tape would damage the skin. Montgomery straps are not placed over open wounds. Mesh gauze would allow air movement.

52. When teaching the client older than age 50 who is receiving long-term prednisone therapy, the nurse should recommend? ■ 1. Take the prednisone with food. ■ 2. Take over-the-counter drugs as needed. ■ 3. Exercise three to four times a week. ■ 4. Eat foods that are low in potassium.

1. Nausea, vomiting, and peptic ulcers are gastrointestinal adverse effects of prednisone, so it is recommended that clients take the prednisone with food. In some instances, the client may be advised to take a prescribed antacid prophylactically. The client should never take over-thecounter drugs without notifying the physician who prescribed the prednisone. The client should ask the physician about the amount and kind of exercise because of the need to establish baseline physical values before starting an exercise program and because of the increased potential for comorbidity with increasing age. The client should eat foods that are high in potassium to prevent hypokalemia.

14. If the client who was admitted for myocardial infarction (MI) develops cardiogenic shock, which characteristic sign should the nurse expect to observe? ■ 1. Oliguria. ■ 2. Bradycardia. ■ 3. Elevated blood pressure. ■ 4. Fever.

1. Oliguria occurs during cardiogenic shock because there is reduced blood fl ow to the kidneys. Typical signs of cardiogenic shock include low blood pressure, rapid and weak pulse, decreased urine output, and signs of diminished blood fl ow to the brain, such as confusion and restlessness. Cardiogenic shock is a serious complication of MI, with a mortality rate approaching 90%. Fever is not a typical sign of cardiogenic shock.

92. In providing care to the client with leukemia who has developed thrombocytopenia, the nurse assesses the most common sites for bleeding. Which of the following is not a common site? ■ 1. Biliary system. ■ 2. Gastrointestinal tract. ■ 3. Brain and meninges. ■ 4. Pulmonary system.

1. The biliary system is not especially prone to hemorrhage. Thrombocytopenia (a low platelet count) leaves the client at risk for a potentially life-threatening spontaneous hemorrhage in the gastrointestinal, respiratory, and intracranial cavities.

A client was admitted to the hospital with a diagnosis of frequent symptomatic premature ventricular contractions (PVCs). After sitting up in a chair for a few minutes, the client complains of feeling lightheaded. On auscultation of the heartbeat, the nurse should anticipate which of the following findings? 1 A regular apical pulse 2 An irregular apical pulse 3 A very slow regular apical pulse 4 A very rapid regular apical pulse

2

A client with unstable ventricular tachycardia (VT) loses consciousness and becomes pulseless after an initial treatment with a dose of lidocaine (Xylocaine) intravenously. The nurse caring for the client would immediately obtain which of the following needed items? 1 A pacemaker 2 A defibrillator 3 A second dose of lidocaine 4 An electrocardiogram machine

2 For the client with VT who becomes pulseless, the physician or qualified advanced cardiac life support personnel immediately defibrillates the client. In the absence of this equipment, cardiopulmonary resuscitation is initiated immediately. Options 1, 3, and 4 are not items that are needed immediately in this situation.

The nurse is assessing a client with a diagnosis of polycythemia vera. Which clinical manifestation would the nurse expect to note in this client? 1 Pallor 2 Hypertension 3 A low hematocrit level 4 Pale mucous membranes

2 Polycythemia vera is a myeloproliferative disease that causes increased blood viscosity and blood volume. Manifestations of polycythemia vera include a ruddy complexion, dusky red mucosa, hypertension, dizziness, headache, and a sense of fullness in the head. Signs of congestive heart failure may also be present. The hematocrit level is usually greater than 54% in men and 49% in women.

55. A client has been on long-term prednisone therapy. The nurse should instruct the client to consume a diet high in which of the following? Select all that apply. ■ 1. Carbohydrate. ■ 2. Protein. ■ 3. Trans fat. ■ 4. Potassium. ■ 5. Calcium. ■ 6. Vitamin D.

2, 4, 5, 6. Adverse effects of prednisone are weight gain, retention of sodium and fl uids with hypertension and cushingoid features, a low serum albumin level, suppressed infl ammatory processes with masked symptoms, and osteoporosis. A diet high in protein, potassium, calcium, and vitamin D is recommended. Carbohydrate would elevate glucose and further compromise a client's immune status. Trans fat does not counteract the adverse effects of steroids such as prednisone.

At 0730 hours, a nurse receives a verbal order for a cardiac catheterization to be completed on a client at 1400 hours. Which action should the nurse initiate first? 1. Initiate NPO (nothing per mouth) status for the client. 2. Teach the client about the procedure. 3. Start an intravenous (IV) infusion of 0.9% NaCl. 4. Ask the client to sign a consent form.

1 A cardiac catheterization is an invasive procedure requiring the client to lie still in a supine position. The client is usually sedated with medication, such as midazolam (Versed®), during the procedure. To avoid aspiration, the client should be NPO 6 to 12 hours prior to the procedure. Because of the time element, NPO status should be initiated first and then teaching should occur. A consent form should be signed after the cardiologist has spoken with the client, and then an IV infusion order would be received.

The nurse is assisting in the care of a client who is to be cardioverted. The nurse plans to set the monophasic defibrillator to which of the following starting energy range levels, depending on the specific physician prescription? 1 50 to 100 joules 2 200 to 250 joules 3 250 to 300 joules 4 350 to 400 joules

1 Cardioversion is synchronized countershock to convert an undesirable rhythm to a stable rhythm. When a client is cardioverted, the defibrillator is charged to the energy level prescribed by the physician. Cardioversion is usually started at 50 to 100 joules. Options 2, 3, and 4 are incorrect and identify energy levels that are too high for cardioversion.

The nurse sends a sputum specimen to the laboratory for culture from a client with suspected active tuberculosis (TB). The results report that Mycobacterium tuberculosis is cultured. How would the nurse correctly analyze these results? 1 The results are positive for active tuberculosis. 2 The results indicate a less virulent strain of tuberculosis. 3 The results are inconclusive until a repeat sputum specimen is sent. 4 The results are unreliable unless the client has also had a positive Mantoux test.

1 Culture of Mycobacterium tuberculosis from sputum or other body secretions or tissue confirms the diagnosis of active tuberculosis. Options 2 and 3 are incorrect statements. The Mantoux test is performed to assist in diagnosing TB but does not confirm active disease.

A client with significant flail chest has arterial blood gases (ABGs) that reveal a PaO2 of 68 and a PaCO2 of 51. Two hours ago the PaO2 was 82 and the PaCO2 was 44. Based on these changes, the nurse obtains which of the following items? 1 Intubation tray 2 Chest tube insertion set 3 Portable chest x-ray machine 4 Injectable lidocaine (Xylocaine)

1 Flail chest occurs from a blunt trauma to the chest. The loose segment from the chest wall becomes paradoxical to the expansion and contraction of the rest of the chest wall. The client with flail chest has painful, rapid, shallow respirations while experiencing severe dyspnea. The effort of breathing and the paradoxical chest movement have the net effect of producing hypoxia and hypercapnia. The client develops respiratory failure and requires intubation and mechanical ventilation, usually with positive end expiratory pressure (PEEP); therefore an intubation tray is necessary.

Intravenous immune globulin (IVIG) therapy is prescribed for a child with idiopathic thrombocytopenic purpura (ITP). The nurse determines that this medication is prescribed for the child to: 1 Increase the number of circulating platelets. 2 Provide immunity to the child against infection. 3 Decrease the production of antiplatelet antibodies. 4 Prevent infection after exposure to communicable diseases.

1 IVIG is usually effective to rapidly increase the platelet count. It is thought to act by interfering with the attachment of antibody-coded platelets to receptors on the macrophage cells of the reticuloendothelial system. Corticosteroids may be prescribed to enhance vascular stability and decrease the production of antiplatelet antibodies. Options 2, 3, and 4 are unrelated to the administration of this medication.

The nurse is caring for a client with multiple myeloma who is receiving intravenous hydration at 100 mL per hour. Which finding indicates a positive response to the treatment plan? 1 Creatinine of 1.0 mg/dL 2 Weight increase of 1 kilogram 3 White blood cell count of 6000/mm3 4 Respirations of 18 breaths per minute

1 Multiple myeloma is a malignant proliferation of plasma cells within the bone. Renal failure is a concern in the client with multiple myeloma. In multiple myeloma, hydration is essential to prevent renal damage resulting from precipitation of protein in the renal tubules and excessive calcium and uric acid in the blood. Creatinine is the most accurate measure of renal status. Options 3 and 4 are unrelated to the subject of hydration. Weight gain is not a positive sign when concerned with renal status.

The nurse is monitoring a client who has received antidysrhythmic therapy for the treatment of premature ventricular contractions (PVCs). The nurse would determine this therapy as being less than optimal if the client's PVCs continued to: 1 Occur in pairs 2 Be unifocal in appearance 3 Be fewer than six per minute 4 Fall after the end of the T wave

1 PVCs are considered dangerous when they are frequent (more than six per minute), occur in pairs or couplets, are multifocal (multiform), or fall on the T wave

Which intervention should a nurse plan to incorporate in the care of a surgical client to decrease the risk of deep venous thrombosis (DVT) and pulmonary embolism (PE)? 1. Use of intermittent compression devices on the lower extremities 2. Administration of heparin intravenously 3. Coughing and deep breathing exercises 4. Isometric leg exercises

1 Recommendations to prevent DVT and PE address the need to improve circulation and counter any states of hypercoagulopathy. Intermittent compression devices improve circulation. While administration of heparin will achieve anticoagulation, a low dose of unfractionated or low-molecular-weight heparin is usually ordered subcutaneous, and not intravenous, administration. Coughing and deep breathing exercises and isometric leg exercises are positive actions but do not decrease the risk for DVT and PE.

A client without history of respiratory disease has experienced sudden onset of chest pain and dyspnea and is diagnosed with pulmonary embolus. The nurse immediately implements which expected prescription for this client? 1 Semi-Fowler's position, oxygen, and morphine sulfate intravenously (IV) 2 Supine position, oxygen, andmeperidine hydrochloride (Demerol) intramuscularly (IM) 3 High Fowler's position, oxygen, and meperidine hydrochloride (Demerol) intravenously (IV) 4 High Fowler's position, oxygen, and two tablets of acetaminophen with codeine (Tylenol #3)

1 Standard therapeutic intervention for the client with pulmonary embolus includes proper positioning, oxygen, and intravenous analgesics. The head of the bed is placed in semi-Fowler's position. High Fowler's is avoided because extreme hip flexure slows venous return from the legs and increases the risk of new thrombi. The supine position will increase the dyspnea that occurs with pulmonary embolism. The usual analgesic of choice is morphine sulfate administered IV. This medication reduces pain, alleviates anxiety, and can diminish congestion of blood in the pulmonary vessels because it causes peripheral venous dilation

83. A client uses a metered-dose inhaler (MDI) to aid in management of his asthma. Which action by the client indicates to the nurse that he needs further instruction regarding its use? Select all that apply. ■ 1. Activation of the MDI is not coordinated with inspiration. ■ 2. The client inspires rapidly when using the MDI. ■ 3. The client holds his breath for 3 seconds after inhaling with the MDI. ■ 4. The client shakes the MDI after use. ■ 5. The client performs puffs in rapid succession.

1, 2, 3, 4, 5. Utilization of an MDI requires coordination between activation and inspiration; deep breaths to ensure that medication is distributed into the lungs, holding the breath for 10 seconds or as long as possible to disperse the medication into the lungs, shaking up the medication in the MDI before use, and a suffi cient amount of time between puffs to provide an adequate amount of inhalation medication.

70. The nurse is developing a care plan for a client with leukemia. The plan should include which of the following? Select all that apply. ■ 1. Monitor temperature and report elevation. ■ 2. Recognize signs and symptoms of infection. ■ 3. Avoid crowds. ■ 4. Maintain integrity of skin and mucous membranes. ■ 5. Take a baby aspirin each day.

1, 2, 3, 4. Nursing care of a client with leukemia includes managing and preventing infection, maintaining integrity of skin and mucous membranes, instituting measures to prevent bleeding, and monitoring for bleeding. Aspirin is an anticoagulant; bleeding tendencies, such as petechiae, ecchymosis, epistaxis, gingival bleeding, and retinal hemorrhages are likely due to thrombocytopenia

67. A 10-year-old client is diagnosed with infectious mononucleosis. Her white blood cell (WBC) count is 19,000/μL. She has a streptococcal throat infection and her spleen is enlarged. She has aching muscles. Which of the following instructions should the nurse include in discharge planning with the client and her parents? Select all that apply. ■ 1. Stay on bed rest until the temperature is normal. ■ 2. Gargle with warm saline while the throat is irritated. ■ 3. Increase intake of fl uids until the infection subsides. ■ 4. Take aspirin as long as the fever and myalgia persist. ■ 5. Avoid contact sports while the spleen is enlarged.

1, 2, 3, 5. The nurse should teach this client to stay on bed rest if she has a fever, gargle with warm saline, and increase her oral fl uids to prevent dehydration from the elevated temperature. The client with an enlarged spleen should avoid contact sports due to the increased risk of injury due to the enlargement. The nurse should tell the client to avoid aspirin if she has a fever because of the risk of Reye's syndrome. Instead, nonsteroidal anti-infl ammatory drugs should be used to treat fever and myalgia.

26. The nurse should instruct a young female adult with sickle cell anemia to do which of the following? Select all that apply. ■ 1. Drink plenty of fl uids when outside in hot weather. ■ 2. Avoid travel to cities where the oxygen level is lower. ■ 3. Be aware that since she is homozygous for HbS, she carries the sickle cell trait. ■ 4. Know that pregnancy with sickle cell disease increases the risk of a crisis. ■ 5. Avoid fl ying on commercial airlines.

1, 2, 4. The nurse should teach the client to drink plenty of fl uids when outside in hot weather to avoid becoming dehydrated. The client should avoid high altitudes, such as mountains, where the oxygen levels are low and may precipitate a sickle cell crisis. The nurse should alert young women with sickle cell anemia that pregnancy increases the risk of a crisis. People who are homozygous for HbS have sickle cell anemia; the heterozygous form is the sickle cell carrier trait. A client with sickle cell anemia may fl y on commercial airlines; the airplane is pressurized for an adequate oxygen level.

40. The nurse is to administer subcutaneous heparin to an older adult. What facts should the nurse keep in mind when administering this dose? Select all that apply. ■ 1. It should be administered in the anterior area of the iliac crest. ■ 2. The onset is immediate. ■ 3. Use a 27G, 5/8″ needle. ■ 4. Cephalosporin potentiates the effects of heparin. ■ 5. Double check the dose with another nurse.

1, 3, 4, 5. Older adults may have little subcutaneous tissue, so the area around the anterior iliac crest is a suitable site for these clients. The nurse should use a 27G, 5/8″ needle. Cephalosporin and penicillin potentiate the effects of heparin. Two nurses should check the dose because a dose error could cause hemorrhage. The onset of heparin is not immediate when given subcutaneously

126. A client with acute respiratory distress syndrome (ARDS) has fi ne crackles at lung bases and the respirations are shallow at a rate of 28 breaths/minute. The client is restless and anxious. In addition to monitoring the arterial blood gas results, the nurse should do which of the following? Select all that apply. ■ 1. Monitor serum creatinine and blood urea nitrogen levels. ■ 2. Administer a sedative. ■ 3. Keep the head of the bed fl at. ■ 4. Administer humidifi ed oxygen. ■ 5. Auscultate the lungs

1, 4, 5. Acute respiratory distress syndrome (ARDS) may cause renal failure and superinfection, so the nurse should monitor urine output and urine chemistries. Treatment of hypoxemia can be complicated because changes in lung tissue leave less pulmonary tissue available for gas exchange, thereby causing inadequate perfusion. Humidifi ed oxygen may be one means of promoting oxygenation. The client has crackles in the lung bases, so the nurse should continue to assess breath sounds. Sedatives should be used with caution in clients with ARDS. The nurse should try other measures to relieve the client's restlessness and anxiety. The head of the bed should be elevated to 30 degrees to promote chest expansion and prevent atelectasis

87. The nurse is teaching the client how to use a metered-dose inhaler (MDI) to administer a corticosteroid. Which of the following client actions indicates that he is using the MDI correctly? Select all that apply. ■ 1. The inhaler is held upright. ■ 2. The head is tilted down while inhaling the medicine. ■ 3. The client waits 5 minutes between puffs. ■ 4. The mouth is rinsed with water following administration. ■ 5. The client lies supine for 15 minutes following administration.

1, 4. The client should shake the inhaler and hold it upright when administering the drug. The head should be tilted back slightly. The client should wait about 1 to 2 minutes between puffs. The mouth should be rinsed following the use of a corticosteroid MDI to decrease the likelihood of developing an oral infection. The client does not need to lie supine; instead, the client will likely to be able to breathe more freely if sitting upright.

68. When developing a discharge plan to manage the care of a client with chronic obstructive pulmonary disease (COPD), the nurse should advise the the client to expect to: ■ 1. Develop respiratory infections easily. ■ 2. Maintain current status. ■ 3. Require less supplemental oxygen. ■ 4. Show permanent improvement.

1. A client with COPD is at high risk for development of respiratory infections. COPD is slowly progressive; therefore, maintaining current status and establishing a goal that the client will require less supplemental oxygen are unrealistic expectations. Treatment may slow progression of the disease, but permanent improvement is highly unlikely.

41. A client has a platelet count of 31,000/μL. The nurse should instruct the client to: ■ 1. Pad sharp surfaces to avoid minor trauma when walking. ■ 2. Assess for spontaneous petechiae in the extremities. ■ 3. Keep the room darkened. ■ 4. Check for blood in the urine.

1. A client with a platelet count of 30,000 to 50,000/μL is susceptible to bruising with minor trauma. Padding areas that the client might bump, scratch, or hit may help prevent minor trauma. A platelet count of 15,000 to 30,000/μL may result in spontaneous petechiae and bruising, especially on the extremities. Safety measures to pad surfaces would still be used, but the focus would be on assessing for new spontaneous petechiae. Keeping the room dark does not help the client with a low platelet count. When the count is lower than 20,000/μL, the client is at risk for spontaneous bleeding from the mucous membranes (oral, nasal, urinary, and rectal) and intracranial bleeding

55. Furosemide is administered intravenously to a client with heart failure. How soon after administration should the nurse begin to see evidence of the drug's desired effect? ■ 1. 5 to 10 minutes. ■ 2. 30 to 60 minutes. ■ 3. 2 to 4 hours. ■ 4. 6 to 8 hours.

1. After intravenous injection of furosemide, diuresis normally begins in about 5 minutes and reaches its peak within about 30 minutes. Medication effects last 2 to 4 hours. When furosemide is given intramuscularly or orally, drug action begins more slowly and lasts longer than when it is given intravenously

61. When giving discharge instructions to the client with vasospastic disorder (Raynaud's phenomenon), the nurse should explain that the expected outcome of taking a beta-adrenergic blocking medication is to control the symptoms by: ■ 1. Decreasing the infl uence of the sympathetic nervous system on the tissues in the hands and feet. ■ 2. Decreasing the pain by producing analgesia. ■ 3. Increasing the blood supply to the affected area. ■ 4. Increasing monoamine oxidase.

1. Beta-adrenergic medications block the beta-adrenergic receptors. Therefore, the expected outcome of the medication is to decrease the infl uence of the sympathetic nervous system on the blood vessels in the hands. Beta-adrenergic blockers have no analgesic effects. Increasing blood supply to the affected area is an indirect effect of beta-adrenergic blockers. They do not increase monoamine oxidase, which does not play a role in Raynaud's disease.

90. A client has sudden, severe pain in his back and chest, accompanied by shortness of breath. The client describes the pain as a "tearing" sensation. The physician suspects the client is experiencing a dissecting aortic aneurysm. The code cart is brought into the room because one complication of a dissecting aneurysm is: ■ 1. Cardiac tamponade. ■ 2. Stroke. ■ 3. Pulmonary edema. ■ 4. Myocardial infarction.

1. Cardiac tamponade is a life-threatening complication of a dissecting thoracic aneurysm. The sudden, painful "tearing" sensation is typically associated with the sudden release of blood, and the client may experience cardiac arrest. Stroke, pulmonary edema, and myocardial infarction are not common complications of a dissecting aneurysm

87. The client with acute lymphocytic leukemia (ALL) is at risk for infection. What should the nurse do? ■ 1. Place the client in a private room. ■ 2. Have the client wear a mask. ■ 3. Have staff wear gowns and gloves. ■ 4. Restrict visitors.

1. Clients with ALL are at risk for infection due to granulocytopenia. The nurse should place the client in a private room. Strict hand-washing procedures should be enforced and will be the most effective way to prevent infection. It is not necessary to have the client wear a mask. The client is not contagious and the staff does not need to wear gloves. The client can have visitors; however, they should be screened for infection and use hand-washing procedures.

31. A client is to receive epoetin (Epogen) injections. What laboratory value should the nurse assess before giving the injection? ■ 1. Hematocrit. ■ 2. Partial thromboplastin time. ■ 3. Hemoglobin concentration. ■ 4. Prothrombin time.

1. Epoetin (Epogen) is a recombinant DNA form of erythropoietin, which stimulates the production of RBCs and therefore causes the hematocrit to rise. The elevation in hematocrit causes an elevation in the blood pressure; therefore, the blood pressure is a vital sign that should be checked. The partial thromboplastin time, hemoglobin level, and prothrombin time are not monitored for this drug

137. Which of the following complications is associated with mechanical ventilation? ■ 1. Gastrointestinal hemorrhage. ■ 2. Immunosuppression. ■ 3. Increased cardiac output. ■ 4. Pulmonary emboli.

1. Gastrointestinal hemorrhage occurs in about 25% of clients receiving prolonged mechanical ventilation because of the development of stress ulcers. Clients who are receiving steroid therapy and those with a previous history of ulcers are most likely to be at risk. Other possible complications include incorrect ventilation, oxygen toxicity, fl uid imbalance, decreased cardiac output, pneumothorax, infection, and atelectasis.

98. When assessing the client with Hodgkin's disease, the nurse should observe the client for which of the following fi ndings? ■ 1. Herpes zoster infections. ■ 2. Discolored teeth. ■ 3. Hemorrhage. ■ 4. Hypercellular immunity.

1. Herpes zoster infections are common in clients with Hodgkin's disease. Discoloring of the teeth is not related to Hodgkin's disease but rather to the ingestion of iron supplements or some antibiotics such as tetracycline. Mild anemia is common in Hodgkin's disease, but the platelet count is not affected until the tumor has invaded the bone marrow. A cellular immunity defect occurs in Hodgkin's disease in which there is little or no reaction to skin sensitivity tests. This is called anergy.

118. For a client with rib fractures and a pneumothorax, the physician prescribes morphine sulfate, 1 to 2 mg/hour, given I.V. as needed for pain. The nursing care goal is to provide adequate pain control so that the client can breathe effectively. Which of the following outcomes would indicate successful achievement of this goal? ■ 1. Pain rating of 0 on a scale of 0 to 10 by the client. ■ 2. Decreased client anxiety. ■ 3. Respiratory rate of 26 breaths/minute. ■ 4. PaO2 of 70 mm Hg.

1. If the client reports no pain, then the objective of adequate pain relief has been met. Decreased anxiety is not related only to pain control; it could also be related to other factors. A respiratory rate of 26 breaths/minute is not within normal limits. A PaO2of 70 mm Hg is not within normal limits

140. Which of the following are expected outcomes for a client with pulmonary disease? ■ 1. A relatively matched ventilation-to-perfusion ratio. ■ 2. A low ventilation-to-perfusion ratio. ■ 3. A high ventilation-to-perfusion ratio. ■ 4. An equal PaO2 and PaCO2 ratio.

1. In the normal lung, the volume of blood perfusing the lungs each minute is approximately equal to the amount of fresh gas that reaches the alveoli each minute. Blood gas analysis evaluates respiratory function; the level of dissolved oxygen (PaO2) should be greater than the level of dissolved carbon dioxide (PaCO2).

The nurse is caring for a client diagnosed with an anterior myocardial infarction 2 days ago. Upon assessment, the nurse identifi es a new systolic murmur at the apex. The nurse should fi rst: ■ 1. Assess for changes in vital signs. ■ 2. Draw an arterial blood gas. ■ 3. Evaluate heart sounds with the client leaning forward. ■ 4. Obtain a 12 Lead electrocardiogram

1. Infarction of the papillary muscles is a potential complication of an MI causing ineffective closure of the mitral valve during systole. Mitral regurgitation results when the left ventricle contracts and blood fl ows backward into the left atrium, which is heard at the fi fth intercostal space, left midclavicular line. The murmur worsens during expiration and in the supine or left-side position. Vital sign changes will refl ect the severity of the sudden drop in cardiac output: decrease in blood pressure, increase in heart rate, and increase in respirations. A 12-lead ECG views the electrical activity of the heart; an echocardiogram views valve function.

92. Which of the following health promotion activities should the nurse include in the discharge teaching plan for a client with asthma? ■ 1. Incorporate physical exercise as tolerated into the daily routine. ■ 2. Monitor peak fl ow numbers after meals and at bedtime. ■ 3. Eliminate stressors in the work and home environment. ■ 4. Use sedatives to ensure uninterrupted sleep at night.

1. Physical exercise is benefi cial and should be incorporated as tolerated into the client's schedule. Peak fl ow numbers should be monitored daily, usually in the morning (before taking medication). Peak fl ow does not need to be monitored after each meal. Stressors in the client's life should be modifi ed but cannot be totally eliminated. Although adequate sleep is important, it is not recommended that sedatives be routinely taken to induce sleep

56. Platelets should not be administered under which of the following conditions? ■ 1. The platelet bag is cold. ■ 2. The platelets are 2 days old. ■ 3. The platelet bag is at room temperature. ■ 4. The platelets are 12 hours old.

1. Platelets cannot survive cold temperatures. The platelets should be stored at room temperature and last for no more than 5 days.

45. The client states she does not understand what causes idiopathic thrombocytopenic purpura (ITP). The nurse provides which of the following explanations? ■ 1. It is believed that the platelets are coated with antibodies and the spleen sees them as foreign bodies. ■ 2. It is believed that the liver identifi es the platelets as foreign bodies. ■ 3. It is now believed that the syndrome is related to an underactive immune system. ■ 4. The cause is unknown.

1. Previously the cause was unknown, but recent research suggests that idiopathic thrombocytopenic purpura occurs when antibody-coated platelets are identifi ed as foreign bodies and destroyed by macrophages in the spleen. It is not an idiosyncratic response and is not related to a depressed immune system. CN

85. A client experiencing a severe asthma attack has the following arterial blood gas: pH 7.33; PCO2 48; PO2 58; HCO3 26. Which of the following orders should the nurse perform fi rst? ■ 1. Albuterol (Proventil) nebulizer. ■ 2. Chest x-ray. ■ 3. Ipratropium (Atrovent) inhaler. ■ 4. Sputum culture.

1. The arterial blood gas reveals a respiratory acidosis with hypoxia. A quick-acting bronchodilator, albuterol, should be administered via nebulizer to improve gas exchange. Ipratropium is a maintenance treatment for bronchospasm that can be used with albuterol. A chest x-ray and sputum sample can be obtained once the client is stable.

130. A client with acute respiratory distress syndrome (ARDS) is on a ventilator. The client's peak inspiratory pressures and spontaneous respiratory rate are increasing, and the PO2 is not improving. Using the SBAR (Situation-Background-Assessment- Recommendation) technique for communication, the nurse calls the physician with the recommendation for: ■ 1. Initiating I.V. sedation. ■ 2. Starting a high-protein diet. ■ 3. Providing pain medication. ■ 4. Increasing the ventilator rate.

1. The client may be fi ghting the ventilator breaths. Sedation is indicated to improve compliance with the ventilator in an attempt to lower peak inspiratory pressures. The workload of breathing does indicate the need for increased protein calories; however, this will not correct the respiratory problems with high pressures and respiratory rate. There is no indication that the client is experiencing pain. Increasing the rate on the ventilator is not indicated with the client's increased spontaneous rate.

Metoprolol (Toprol XL) is added to the pharmacologic therapy of a diabetic female diagnosed with stage 2 hypertension initially treated with Furosemide (Lasix) and Ramipril (Altace). An expected therapeutic effect is: ■ 1. Decrease in heart rate. ■ 2. Lessening of fatigue. ■ 3. Improvement in blood sugar levels. ■ 4. Increase in urine output.

1. The effect of a beta blocker is a decrease in heart rate, contractility, and afterload, which leads to a decrease in blood pressure. The client at fi rst may have an increase in fatigue when starting the beta blocker. The mechanism of action does not improve blood sugar or urine output.

17. A client returned home from an overseas tour of duty and tells the nurse he is always tired. He has a temperature of 99.5° F (37.5° C). His skin is dark bronze, and his urine has a dark color. His hemoglobin level is 9 g/dL; his hematocrit is 49, and red blood cells are 2.75 million/μL. What should the nurse do fi rst? ■ 1. Initiate an intake and output record. ■ 2. Place the client on bed rest. ■ 3. Place the client on contact isolation. ■ 4. Keep the client out of sunlight.

1. The nurse should prepare to start an intake and output record because the client is exhibiting clinical manifestations of anemia with jaundice and is demonstrating a fl uid imbalance. The client does not need to be on bed rest at this point. The client is not contagious and does not need to be placed in contact isolation. The changes in the color of the skin and urine are related to the jaundice and will not be affected by sunlight.

134. Which of the following conditions can place a client at risk for acute respiratory distress syndrome (ARDS)? ■ 1. Septic shock. ■ 2. Chronic obstructive pulmonary disease. ■ 3. Asthma. ■ 4. Heart failure.

1. The two risk factors most commonly associated with the development of ARDS are gramnegative septic shock and gastric content aspiration. Nurses should be particularly vigilant in assessing a client for onset of ARDS if the client has experienced direct lung trauma or a systemic infl ammatory response syndrome (which can be caused by any physiologic insult that leads to widespread infl ammation). Chronic obstructive pulmonary disease, asthma, and heart failure are not direct causes of ARDS.

client with symptoms of anemia and a hemoglobin of 7.8 g/dL refuses blood and blood products transfusions for religious reasons. A nurse should anticipate that a health-care provider might prescribe: SELECT ALL THAT APPLY. 1. Epoetin alfa (Procrit®) 2. Folic acid 3. Albumin 4. Platelets 5. Fresh frozen plasma 6. Granulocytes

12 Epoetin alfa (erythropoietin growth factor) and folic acid promote erythropoiesis (production of red blood cells), thus decreasing the need for transfusions. Folic acid also stimulates production of white blood cells and platelets. According to the evidence base, for persons with hemoglobin less than 8 g/dL, the use of either transfusion or erythropoietic growth factor was rated "appropriate." Albumin, platelets, plasma, and granulocytes are all blood products.

A client with an abdominal aortic aneurysm is having a high resolution computed tomography (CT) scan to determine the feasibility for an endovascular repair. Which collaborative interventions should a nurse anticipate to decrease the client's likelihood of developing nephrotoxicity? SELECT ALL THAT APPLY. 1. Administration of sodium bicarbonate 1 hour before injection of the intravenous (IV) contrast dye 2. Administration of 0.9% NaCl at 100 mL per hour before and after the CT scan 3. Administration of acetylcysteine (Mucomyst®) orally before and after the study 4. Monitoring aPTT level before and after the CT scan 5. Placing the client on a low potassium diet

123 The contrast dye used in a high resolution presents a risk to the client's renal function. Intravenous fluids are administered to maintain hydration and enhance excretion of the dye. Sodium bicarbonate is administered 1 hour before and 6 hours after the IV contrast dye. Acetylcysteine is administered orally the day before and the day of the CT scan to prevent acute renal failure. Both sodium bicarbonate and acetylcysteine are free-radical scavengers that sequester the contract by-products that are destructive to renal cells. The aPTT (activated thromboplastin time) is a measure of blood coagulation and is not related to nephrotoxicity. A low potassium diet may be ordered for a client with renal failure but is not a preventive measure.

A client is hospitalized with a diagnosis of sickle cell crisis. Which findings should lead a nurse to conclude that outcomes have been achieved for this client? SELECT ALL THAT APPLY. 1. Leukocyte count 7,500/mm3 2. Describes the importance of keeping warm 3. Acute pain controlled at less than 3 on a 0 to 10 scale with analgesics 4. Free of chest pain or dyspnea 5. Blood transfusions effective in diminishing cell sickling 6. Hydroxyurea (Hydrea®) effective in suppressing leukocyte formation

1234 A leukocyte count of 7,500/mm3 is within normal range (5,000 to 10,000/mm3 indicates the absence of an infection). Keeping warm and avoiding chills will help to prevent infection. Also, cold causes vasoconstriction, slowing blood flow and aggravating the sickling process. Acute pain is due to tissue hypoxia from the agglutination of sickled cells within blood vessels. Acute chest syndrome and pulmonary hypertension are two of the many complications associated with sickle cell disease. Red blood cell transfusions may help to prevent complications, but transfusions do not alter the person's body from producing the deformed erythrocytes. Hydroxyurea can decrease the permanent formation of sickled cells. A side effect (not therapeutic effect) of Hydroxyurea is suppression of leukocyte formation.

16. A client is admitted from the emergency department after falling down a fl ight of stairs at home. Her vital signs are stable and her history states that she had a gastric stapling 2 years ago and takes neomycin for acne. The client jokes about how she is clumsy lately and trips over things. The nurse should ask the client which of the following questions? Select all that apply. ■ 1. "Are you experiencing numbness in your extremities?" ■ 2. "How much vitamin B12 are you getting?" ■ 3. "Are you feeling depressed?" ■ 4. "Do you feel safe at home?" ■ 5. "Are you getting suffi cient iron in your diet?"

1234 The nurse should ask the client about symptoms related to pernicious anemia because she had her stomach stapled 2 years ago and shows no history of supplemental vitamin B12. Numbness and tingling relate to a loss of intrinsic factor from the gastric stapling. Intrinsic factor is necessary for absorption of vitamin B12. The nurse should suspect pernicious anemia if the client is not taking supplemental vitamin B12. Other signs and symptoms of pernicious anemia include cognitive problems and depression. The nurse also should ask about the client's support at home in case the fall was not an accident. Pernicious anemia is not related to dietary intake of iron.

A client is admitted with a diagnosis of acute infective endocarditis (IE). Which findings during a nursing assessment support this diagnosis? SELECT ALL THAT APPLY. 1. Skin petechiae 2. Crackles in lung bases 3. Peripheral edema 4. Murmur 5. Arthralgia 6. Decreased erythrocyte sedimentation rate (ESR)

12345 Vegetations that adhere to the heart valves can break off into the circulation, causing embolism, valve incompetence, and a murmur. A vascular sign of microembolism is skin petechiae. Crackles and peripheral edema occur due to heart failure secondary to IE. Arthralgia (joint pain) can occur from microembolism and inadequate perfusion. The ESR (rate at which red blood cells settle) should increase, not decrease, during an inflammatory process.

A nurse is caring for multiple 25-year-old female clients. For which clients should the nurse plan to obtain a referral for genetic counseling and family planning? SELECT ALL THAT APPLY. 1. Client diagnosed with thalassemia major 2. Client diagnosed with sickle cell disease 3. Client diagnosed with hemophilia A 4. Client diagnosed with autoimmune hemolytic anemia 5. Client diagnosed with hemophilia B

1235

The nurse is planning care for a client diagnosed with deep vein thrombosis (DVT) of the left leg who is experiencing severe edema and pain in the affected extremity. Which intervention(s) should the nurse plan to include in the care of this client? Select all that apply. r 1 Elevate the left leg. r 2 Apply moist heat to the left leg. r 3 Administer acetaminophen (Tylenol). r 4 Ambulate in the hall three times per shift. r 5 Administer anticoagulation as prescribed.

1235 Management of the client with DVT who is experiencing severe edema and pain includes bed rest; limb elevation; relief of discomfort with warm moist heat and analgesics as needed; anticoagulant therapy; and monitoring for signs of pulmonary embolism. In current practice, activity restriction may not be prescribed if the client is receiving low-molecularweight anticoagulation; however, some physicians may still prefer bed rest for the client.

The nurse is developing a plan of care for a client with a dissecting abdominal aortic aneurysm. Which interventions should be included in the plan of care? Select all that apply. r 1 Assess peripheral circulation r 2 Monitor for abdominal distention. r 3 Tell the client that abdominal pain is expected r 4 Turn the client to the side to look for ecchymoses on the lower back r 5 Perform deep palpation of the abdomen to assess the size of the aneurysm

124 If the client has an abdominal aortic aneurysm, the nurse is concerned about rupture and monitors the client closely. The nurse tells the client to report abdominal pain, or back pain, which may radiate to the groin, buttocks, or legs because this is a sign of rupture. The nurse also avoids deep palpation in the client in whom a dissecting abdominal aortic aneurysm is known or suspected. Doing so could place the client at risk for rupture. The nurse should assess peripheral circulation and monitor for abdominal distention. The nurse also looks for ecchymoses on the lower back to determine if the aneurysm is leaking.

A nurse explains to another nurse that chronic lymphocytic leukemia (CLL) is: SELECT ALL THAT APPLY. 1. a malignancy of activated B lymphocytes. 2. the most common malignancy of older adults. 3. unresponsive to chemotherapy treatment. 4. often not treated in its early stages but the client is monitored. 5. an excessive accumulation of immature lymphocytes in the bone marrow. 6. often asymptomatic and diagnosed incidentally during routine physical examination.

1246 CLL derives from a malignant clone of B lymphocytes. T-lymphocytic CLL is rare. Two-thirds of all persons with CLL are older than 60 years at diagnosis. Treatment is initiated when symptoms are severe (night sweats, painful lymphadenopathy) or the disease progresses to later stages. Because many persons are asymptomatic, it is often diagnosed during a routine physical or treatment for another condition. Treatment for CLL includes chemotherapy with fludarabine (Fludara®), but a major side effect is prolonged bone marrow suppression. In CLL there is an accumulation of mature-appearing but functionally inactive lymphocytes. Excessive accumulation of immature lymphocytes occurs in acute lymphocytic leukemia (ALL). ALL after 15 years of age is relatively uncommon. Be older and mature" (B = B lymphocytes; o = older adult; a = asymptomatic; m = mature but inactive lymphocytes).

A client with symptoms of intermittent claudication receives treatment with a peripheral percutaneous transluminal angioplasty procedure with placement of an endovascular stent. During a follow- up home visit, a nurse determines that the client is making lifestyle changes to decrease the likelihood of re-stenosis and arterial occlusion. Which observations of the client's actions support this conclusion? SELECT ALL THAT APPLY. 1. States participating in an exercise program 2. Abstaining from nicotine 3. Wearing support hose 4. States receiving foot care from a podiatrist 5. Following a low saturated fat diet 6. Taking the medication rosuvastatin calcium (Crestor®)

1256 Reducing client risk factors in order to slow the arteriosclerotic process may delay progression of the disease. Exercising promotes collateral circulation. Smoking cessation, following a low saturated fat diet, and taking medications to lower cholesterol also deter the arteriosclerotic process. There is evidence that more than 30% of clients having procedures to improve claudication seek further intervention within a year. Wearing support hose may impede circulation. Receiving professional foot care is a positive factor but does not prevent the progressive nature of peripheral arterial disease.

A client has undergone a vaginal hysterectomy. The nurse writes which appropriate interventions on the client's nursing care plan to decrease the risk of deep vein thrombosis or thrombophlebitis? Select all that apply. r 1 Using pneumatic compression boots r 2 Maintaining bedrest for 24 to 48 hours r 3 Assisting with range of motion leg exercises r 4 Applying antiembolism stockings and removing them twice daily for assessment r 5 Elevating the knees with the knee gatch on the bed

134 The client is at risk for deep vein thrombosis or thrombophlebitis after this surgery, as for any other major surgery. For this reason, the nurse implements measures that will prevent this complication. Ambulation, pneumatic compression boots, range of motion exercises, and antiembolism stockings are all helpful. The nurse should avoid elevating the knees using the knee gatch in the bed, which inhibits venous return, and places the client more at risk for deep vein thrombosis or thrombophlebitis.

A nurse is preparing a client for a thoracic aneurysm repair. Which assessment findings lead the nurse to suspect that a rupture has occurred? SELECT ALL THAT APPLY. 1. Severe chest pain radiating to the back 2. Abdominal distention 3. Hypotension 4. Dyspnea 5. Oliguria

1345 A thoracic aneurysm that ruptures will cause pain in the thoracic area. Blood loss will lead to low blood pressure and scant urinary output. The pressure from the hemorrhage will interfere with the client's breathing. A thoracic aneurysm does not cause abdominal distention because the bleeding is in the thoracic area.

\. The nurse is monitoring a client who is receiving a blood transfusion when the client complains of diaphoresis, warmth, and a backache. The nurse suspects a transfusion reaction and should take which actions? Select all that apply. r 1 Contact the physician. r 2 Remove the IV catheter. r 3 Document the occurrence. r 4 Stop the blood transfusion. r 5 Hang 0.9% sodium chloride solution

1345 If a client experiences a transfusion reaction, the nurse stops the transfusion and prevents the infusion of any additional blood; then the nurse hangs a bag of 0.9% sodium chloride solution. This maintains IV access and helps maintain the client's intravascular volume. To preserve the IV access, the nurse should not discontinue the IV site. The physician is notified, as is the blood bank. The nurse follows agency procedures with regard to additional actions to take, such as obtaining a urine specimen and returning the blood bag and tubing to the blood bank. The nurse also documents the occurrence, the actions taken, and the client's response

A client with a diagnosis of chronic obstructive pulmonary disease (COPD) has developed polycythemia vera, and a nurse has completed teaching on measures to prevent complications. During a home health visit, the nurse evaluates that the client is correctly following the teaching when the client: SELECT ALL THAT APPLY. 1. tells the nurse about discontinuing iron supplements. 2. relays increasing alcohol intake to decrease blood viscosity. 3. records the amount consumed after drinking a glass of water. 4. discusses yesterday's phlebotomy treatment to remove blood. 5. shows the nurse a menu plan for eating three large meals daily. 6. reclines in a recliner chair with legs uncrossed, wearing antiembolic stockings (TEDS®).

1346 Iron supplements, including those in multivitamins, should be avoided because the iron stimulates red blood cell production. Increasing fluid intake to 3,000 mL daily will help decrease blood viscosity. Phlebotomy is performed on a routine or intermittent basis to diminish blood viscosity, deplete iron stores, and decrease the client's ability to manufacture excess erythrocytes. Elevating the legs, avoiding constriction or crossing the legs, and wearing antiembolic socks help prevent deep vein thrombosis. Alcohol increases the risk of bleeding. Frequent, small meals are better tolerated, especially if the liver is involved.

nurse is planning care for a client admitted with a new diagnosis of persistent atrial fibrillation with rapid ventricular response. Although the client has had no previous cardiac problems, the client has been in atrial fibrillation for more than 2 days. The nurse should anticipate that the health-care provider is likely to initially order: SELECT ALL THAT APPLY. 1. oxygen. 2. immediate cardioversion. 3. administration of amiodarone (Cordarone®). 4. initiation of a IV heparin infusion. 5. immediate catheter-directed ablation of the AV node. 6. administration of a calcium channel antagonist such as diltiazem (Cardizem®).

1346 The ineffective atrial contractions or loss of atrial kick with atrial fibrillation can decrease cardiac output. Administering oxygen enhances tissue oxygenation. Amiodarone is used for pharmacological cardioversion of the atrial fibrillation rhythm. The client is at risk for thrombi in the atria from stasis. Anticoagulant therapy is used to prevent thromboembolism. Diltiazem, a calcium channel antagonist, is prescribed to slow the ventricular response to atrial fibrillation. An alternative to a calcium channel antagonist would be the use of a beta blocker, such as esmolol, metoprolol, or propranolol. Cardioversion would only be considered if medications were ineffective in converting the client's rhythm and only after the presence of an atrial clot has been ruled out. Ablation of the AV node would only be considered if medications were ineffective in controlling the client's heart rate.

A nurse is preparing to perform a Mantoux skin test. Which interventions apply in relation to this test? Select all that apply. r 1 Explain the procedure to the client. r 2 Obtain a 3-mL syringe with a ½-inch needle for the injection. r 3 Mark the test area to locate it for reading 48 to 72 hours after injection. r 4 Bunch up the skin and insert the needle with the needle bevel facing downward. r 5 Ask the client about a previous history of a positive purified protein derivative (PPD) reaction. r 6 Cleanse the injection site on the lower dorsal surface of the forearm with alcohol and allow it to dry.

1356q The nurse would always explain the procedure to the client and then assess him or her for a previous history of a PPD reaction. The test should not be administered if the client has such a history. The nurse would use a tuberculin syringe (not a 3-mL syringe) with a ½-inch 26- or 27-gauge needle. The injection site on the lower dorsal surface of the forearm is cleansed with alcohol and allowed to dry. The skin is stretched taut and 0.1 mL of solution containing 0.5 tuberculin units of PPD is injected. The injection is made just under the surface of the skin with the needle bevel facing upward to provide a discrete elevation of the skin (a wheal) 6 to 10 mm in diameter. The test area is marked to locate it for reading and the test area is read 48 to 72 hours after injection.

90. A nurse is teaching a client to use a metereddose inhaler (MDI) to administer his bronchodilator medication. Indicate the correct order of the steps the client should take to use the MDI appropriately. 1. Shake the inhaler immediately before use 2. Hold breath for 5 to 10 seconds and then exhale. 3. Activate the MDI on inhalation. 4. Breathe out through the mouth.

1432 When using inhalers, clients should fi rst shake the inhaler to activate the MDI, and then breathe out through the mouth. Next, the client should activate the MDI while inhaling, hold the breath for 5 to 10 seconds, and then exhale normally.

A nurse discusses the self-care guidelines to minimize the side effects of radiation on the skin. Which actions, to reduce radiation skin reactions, should the nurse explain to the client? SELECT ALL THAT APPLY. 1. Wear loose-fitting, soft clothing over the treated skin. 2. Use a straightedged razor to shave the hair in the treated area. 3. Swim only in swimming pools to avoid stagnant water. 4. Use only skin-care products suggested by the radiation staff. 5. Apply skin products immediately before radiation treatment. 6. Wash treated area gently with lukewarm water and mild soap.

146 Wearing loose-fitting, soft clothing over the treated skin, using only skin-care products suggested by the radiation staff, and washing the treated area gently with lukewarm water and mild soap are recommended skin-care activities to reduce radiation skin reactions. The use of electric razors for shaving a treated area is recommended. Clients are advised to avoid swimming in chlorinated water and to delay the application of skin-care products within 4 hours of radiation treatment.

A nurse receives a serum laboratory report for six different clients with admitting diagnoses of chest pain. After reviewing all of the lab reports, in which order should the nurse address each lab value? Prioritize the order in which the nurse should address each of the clients' results. ______ Troponin T 42 ng/mL (0.0-0.4 ng/mL) ______ WBC 11,000 K/μL ______ Hgb 7.2 g/dL ______ SCr 2.2 mg/dL ______ K 2.2 mEq/L ______ Total cholesterol 430 mg/dL

163425 The nurse should address the elevated troponin level first. Cardiospecific troponins (troponin T, cTnT, and troponin I, cTnI) are released into circulation after myocardial injury and are highly specific indicators of myocardial infarction. Since "time is muscle," the client needs to be treated immediately to prevent extension of the infarct and possible death. The nurse should address the decreased serum potassium level (K) second. The normal serum K level is 3.5 to 5.8 mEq/L. A low serum K level can cause life-threatening dysrhythmias. The normal hemoglobin (Hgb) is 13.1 to 17.1 g/dL. A low Hgb can contribute to inadequate tissue perfusion and contribute to myocardial ischemia. The normal serum creatinine (SCr) is 0.4 to 1.4 mg/dL. Impaired circulation may be causing this alteration and further client assessment is needed. Medication doses may need to be adjusted with impaired renal perfusion. The normal total serum cholesterol should be less than 200 mg/dL. This is a risk factor for development of coronary artery disease. The client needs teaching. The normal white blood cell (WBC) count is 3.9 to 11.9 K/μL. Because the finding is normal, it can be addressed last.

A client admitted with unstable angina is started on intravenous heparin and nitroglycerin. The client's chest pain resolves, and the client is weaned from the nitroglycerin. Noting that the client had a synthetic valve replacement for aortic stenosis 2 years ago, a physician writes an order to restart the oral warfarin (Coumadin®) 5 mg at 1900 hours. Which is the nurse's best action? 1. Administer the warfarin as prescribed. 2. Call the physician to question the warfarin order. 3. Discontinue the heparin drip and then administer the warfarin. 4. Hold the dose of warfarin until the heparin has been discontinued.

1Both heparin and warfarin are anticoagulants, but their actions are different. Oral warfarin requires 3 to 5 days to reach effective levels. It is usually begun while the client is still on heparin. Calling the physician is unnecessary. The nurse's scope of practice does not permit altering medication orders. The nurse should neither discontinue the heparin nor hold the warfarin without a written order.

A nurse is caring for a client following a coronary artery bypass graft. Which assessment finding in the immediate postoperative period should be most concerning to the nurse? 1. No chest tube output for 1 hour when previously it was copious 2. Client temperature of 99.1°F (37.2°C) 3. Arterial blood gas (ABG) results show pH 7.32; Pco2 48; HCO3 28; Po2 80 4. Urine output of 160 mL in the last 4 hours

1\ A copiously draining chest tube that is no longer draining indicates an obstruction. There is an increased risk for cardiac tamponade or pleural effusion. A slight elevation in temperature could be the effects of rewarming after surgery. This should continue to be monitored, but is not immediately concerning. The ABG results show compensated respiratory acidosis. Though the pH is low and the PCO2 is high, the kidneys are compensating by conserving bicarbonate (HCO3). Normal pH is 7.35-7.45, PCO2 32-42 mm Hg, HCO3 20-24 mmol/L, and PO2 75-100 mm Hg. A urine output of 160 mL/4 hr is equivalent to 40 mL/hr; adequate, but it warrants continued monitoring. Less than 30 mL/hr indicates decreased renal function.

A nurse is caring for a client with a left-sided chest tube attached to a wet suction chest tube system. Which observation by the nurse would require immediate intervention? 1. Bubbling in the suction chamber 2. Dependent loop hanging off the edge of the bed 3. Banded connections between tubing sections 4. Occlusive dressing over chest tube insertion site

2 A dependent loop creates pressure back up and prevents fluid from draining; this requires immediate intervention to prevent lung collapse. Bubbling in a wet suction chest tube system indicates that the suction is working and is an expected finding as are banded connections between sections of tubing. An occlusive dressing helps to prevent air from leaking into the subcutaneous space and maintains integrity of the closed drainage system.

The nurse notes an isolated premature ventricular contraction (PVC) on the cardiac monitor. The appropriate nursing action is to: 1 Prepare for defibrillation. 2 Continue to monitor the rhythm. 3 Notify the physician immediately. 4 Prepare to administer lidocaine hydrochloride (Xylocaine).

2 As an isolated occurrence, the PVC is not life threatening. In this situation, the nurse should continue to monitor the client. Frequent PVCs, however, may be precursors of more life-threatening rhythms, such as ventricular tachycardia and ventricular fibrillation. If this occurs, the physician needs to be notified. Defibrillation is done to treat ventricular fibrillation. Lidocaine hydrochloride is not needed to treat isolated PVCs; it may be used to treat frequent PVCs in a client who is symptomatic and is experiencing decreased cardiac output.

A client diagnosed with Hodgkin's lymphoma develops radiation pneumonitis 3 months after radiation treatment. For which symptoms of radiation pneumonitis should a nurse observe the client? 1. Tachypnea, hypotension, and fever 2. Cough, fever, and dyspnea 3. Bradypnea, cough, and decreased urine output 4. Cough, tachycardia, and altered mental status

2 Cough, fever, and dyspnea are classic symptoms in radiation pneumonitis due to a decrease in the surfactant in the lung. Hypotension, decreased urine output, and altered mental status are symptoms that are not common in radiation pneumonitis.

nurse is preparing to teach the parents of a child with anemia about the dietary sources of iron that are easy for the body to absorb. Which food item should the nurse include in the teaching plan? 1 Fruits 2 Poultry 3 Apricots 4 Vegetables

2 Dietary sources of iron that are easy for the body to absorb include meat, poultry, and fish. Vegetables, fruits, cereals, and breads are also dietary sources of iron, but they are harder for the body to absorb

A client in labor has a concurrent diagnosis of sickle cell anemia. Which action has priority to assist in preventing a sickling crisis from occurring during labor? 1 Reassuring the client 2 Administering oxygen 3 Preventing bearing down 4 Maintaining strict asepsis

2 During the labor process, the client with sickle cell anemia is at high risk for being unable to meet the oxygen demands of labor. Administering oxygen will prevent sickle cell crisis during labor. Options 1 and 4 are appropriate actions but are unrelated to sickle cell crisis. Option 3 is inappropriate.

client who has no history of immunosuppressive disease and is at low risk for tuberculosis has a Mantoux test. The results indicate an area of induration that is 8 mm in size. The nurse interprets that the client: 1 Has active tuberculosis 2 Has a negative response 3 Has a history of tuberculosis 4 Has been exposed to tuberculosis

2 Induration of 15mmormore is considered positive for clients in low-risk groups. More than 5 mm of induration is considered a positive result for clients with known or suspected human immunodeficiency virus infection, persons with organ transplants, people in close contact with a known case of tuberculosis, and those with a chest x-ray study suggestive of previous tuberculosis.More than 10mmof induration is considered positive in all other high-risk groups, such as intravenous drug users.

A client's laboratory test results reveal a decreased serum transferrin and total ironbinding capacity (TIBC). Which disorder is the most likely cause of the client's anemia? 1 Infection 2 Malnutrition 3 Iron deficiency 4 Sickle cell disease

2 Malnutrition can cause reductions in the serum transferrin and the TIBC. Infection is an unrelated option. Sickle cell anemia is diagnosed by determining that the client has hemoglobin S. Iron-deficiency anemia is usually characterized by decreased iron-binding capacity but increased transferrin levels. Additionally, in clinical practice, the hemoglobin level is routinely used to detect iron-deficiency anemia.

A client is diagnosed with pernicious anemia. The nurse reviews the client's health history for disorders involving which organ responsible for vitamin B12 absorption? 1 Liver 2 Ileum 3 Hepatobiliary 4 Gastrointestinal

2 Pernicious anemia can occur in a client who has a disease involving the ileum, where vitamin B12 is absorbed. The nurse checks the client's history for small bowel disorders to detect this risk factor. The liver is not usually related to impaired B12 absorption. Hepatobiliary refers to the liver and gallbladder, and gastrointestinal refers to the organ systems that include organs such as the stomach, liver, gallbladder, and ileum.

A client with Raynaud's disease is seen in a vascular clinic 6 weeks after nifedipine (Procardia®) has been prescribed. A nurse evaluates that the medication has been effective when which findings are noted? 1. The client's blood pressure is 110/68 mm Hg. 2. The client states experiencing less pain and numbness. 3. The client states that tolerance to heat is improved. 4. The client walks without claudication

2 Raynaud's disease is a disease in which cutaneous arteries in the extremities have recurrent episodes of vasospasm with blanching and then redness. The episodes are brought on by cold and result in pain and numbness. Nifedipine, a calcium channel blocker, causes vasodilation, which reduces pain and numbness. Nifedipine is used as an antihypertensive agent but that is not the purpose here. The client is at risk to develop hypotension as an adverse effect. Tolerance to cold, not heat, should improve. Claudication is not associated with Raynaud's disease but is associated with arteriosclerotic changes in the larger arteries.

A nurse is reviewing the serum laboratory test results for a client with sickle cell anemia. Which parameter does the nurse anticipate will be elevated? 1 Sodium 2 Hemoglobin-S 3 Hemoglobin A1c 4 Prothrombin time

2 Sickle cell anemia is a severe anemia that affects African Americans predominantly and is characterized by sickled hemoglobin, or Hgb-S. The client must have two abnormal genes yielding hemoglobin- S to have sickle cell anemia. A client could have sickle cell trait by carrying one hemoglobin-A gene and one hemoglobin-S gene; then, the client has a less severe form of sickle cell anemia. Options 1, 3, and 4 are unrelated to sickle cell anemia

The nurse is administering epoetin alfa (Epogen) to a client with chronic renal failure. The nurse monitors the client for which adverse effect of this therapy? 1 Anemia 2 Hypertension 3 Iron intoxication 4 Bleeding tendencies

2 The client taking epoetin alfa is at risk of hypertension and seizure activity as the most serious adverse effects of therapy. This medication is used to treat anemia. The medication does not cause iron intoxication. Bleeding tendencies is not an adverse effect of this medication.

A nurse is caring for multiple clients on a medical unit. Which client, who has been diagnosed with a lower extremity deep venous thrombosis (DVT), should the nurse plan for possible placement of a filter in the inferior vena cava to protect against pulmonary embolism? 1. A 22-year-old female who has been taking oral contraceptives 2. A 65-year-old client admitted with a bleeding gastric ulcer 3. A 55-year-old client who had a total knee joint replacement 4. A 52-year-old female who had a vaginal hysterectomy 6 weeks earlier

2 The client with the bleeding gastric ulcer is not a candidate for anticoagulant therapy and, therefore, needs the inferior vena cava filter to prevent an embolus from the DVT reaching the pulmonary circulation. The other clients have no contraindications listed for anticoagulant therapy.

37. The nurse is teaching a client with polycythemia vera about potential complications from this disease. Which manifestations should the nurse include in the client's teaching plan? Select all that apply. ■ 1. Hearing loss. ■ 2. Visual disturbance. ■ 3. Headache. ■ 4. Orthopnea. ■ 5. Gout. ■ 6. Weight loss

2, 3, 4, 5. Polycythemia vera, a condition in which too many RBCs are produced in the blood serum, can lead to an increase in the hematocrit and hypervolemia, hyperviscosity, and hypertension. Subsequently, the client can experience dizziness, tinnitus, visual disturbances, headaches, or a feeling of fullness in the head. The client may also experience cardiovascular symptoms such as heart failure (shortness of breath and orthopnea) and increased clotting time or symptoms of an increased uric acid level such as painful, swollen joints (usually the big toe). Hearing loss and weight loss are not manifestations associated with polycythemia vera.

29. Which safety measures would be most important to implement when caring for a client who is receiving 2 units of packed red blood cells (PRBCs)? Select all that apply. ■ 1. Verify that the ABO and Rh of the 2 units are the same. ■ 2. Infuse a unit of PRBCs in less than 4 hours. ■ 3. Stop the transfusion if a reaction occurs, but keep the line open. ■ 4. Take vital signs every 15 minutes while the unit is transfusing. ■ 5. Inspect the blood bag for leaks, abnormal color, and clots. ■ 6. Use a 22-gauge catheter for optimal fl ow of a blood transfusion.

2, 3, 5. The American Association of Blood Banks recommends that two qualifi ed people, such as two registered nurses or a physician and a registered nurse, compare the name and number on the identifi cation bracelet with the tag on the blood bag. Verifying that the two units are the same is not a recommendation. Rather, the verifi cation is always with the client, not with bags of blood. A unit of blood should infuse in 4 hours or less to avoid the risk of septicemia since no preservatives are used. When a blood transfusion reaction occurs, the blood transfusion should be stopped immediately, but the I.V. line should be kept open so that emergency medications and fl uids can be administered. The unit of PRBCs should be inspected for contamination by looking for leaks, abnormal color, clots, and excessive air bubbles. When a unit of PRBCs is being transfused, vital signs are assessed before the transfusion begins, after the fi rst 15 minutes, and then every hour until 1 hour after the transfusion has been completed. When PRBCs are being administered, a 20-gauge or larger needle is needed to avoid destroying the RBCs passing through the lumen and to allow for maximal fl ow rate.

64. A client who has been diagnosed with tuberculosis has been placed on drug therapy. The medication regimen includes rifampin (Rifadin). Which of the following instructions should the nurse include in the client's teaching plan related to the potential adverse effects of rifampin? Select all that apply. ■ 1. Having eye examinations every 6 months. ■ 2. Maintaining follow-up monitoring of liver enzymes. ■ 3. Decreasing protein intake in the diet. ■ 4. Avoiding alcohol intake. ■ 5. The urine may have an orange color.

2, 4, 5. A potential adverse effect of rifampin (Rifadin) is hepatotoxicity. Clients should be instructed to avoid alcohol intake while taking rifampin and keep follow-up appointments for periodic monitoring of liver enzyme levels to detect liver toxicity. Rifampin causes the urine to turn an orange color and the client should understand that this is normal. It is not necessary to restrict protein intake in the diet or have the eyes examined due to rifampin therapy.

57. The nurse is teaching a client who has been diagnosed with tuberculosis how to avoid spreading the disease to family members. Which statement(s) by the client indicate(s) that he has understood the nurse's instructions? Select all that apply. ■ 1. "I will need to dispose of my old clothing when I return home." ■ 2. "I should always cover my mouth and nose when sneezing." ■ 3. "It is important that I isolate myself from family when possible." ■ 4. "I should use paper tissues to cough in and dispose of them promptly." ■ 5. "I can use regular plates and utensils whenever I eat."

2, 4, 5. When teaching the client how to avoid the transmission of tubercle bacilli, it is important for the client to understand that the organism is transmitted by droplet infection. Therefore, covering the mouth and nose when sneezing, using paper tissues to cough in with prompt disposal, and using regular plates and utensils indicate that the client has understood the nurse's instructions about preventing the spread of airborne droplets. It is not essential to discard clothing, nor does the client need to isolate himself from family members.

103. The nurse explains to the client with Hodgkin's disease that a bone marrow biopsy will be taken after the aspiration. What should the nurse explain about the biopsy? ■ 1. "Your biopsy will be performed before the aspiration because enough tissue may be obtained so that you won't have to go through the aspiration." ■ 2. "You will feel a pressure sensation when the biopsy is taken but should not feel actual pain; if you do, tell the doctor so that you can be given extra numbing medicine." ■ 3. "You may hear a crunch as the needle passes through the bone, but when the biopsy is taken, you will feel a suction-type pain that will last for just a moment." ■ 4. "You will be shaved and cleaned with an antiseptic agent, after which the doctor will inject a needle without making an incision to aspirate out the bone marrow."

2. A biopsy needle is inserted through a separate incision in the anesthetized area. The client will feel a pressure sensation when the biopsy is taken but should not feel actual pain. The client should be instructed to inform the physician if pain is felt so that more anesthetic agent can be administered to keep the client comfortable. The biopsy is performed after the aspiration and from a slightly different site so that the tissue is not disturbed by either test. The client will feel a suction-type pain for a moment when the aspiration is being performed, not the biopsy. A small incision is made for the biopsy to accommodate the larger-bore needle. This may require a stitch.

71. A client with neutropenia has an absolute neutrophil count of 900. What is the client's risk of infection? ■ 1. Normal risk. ■ 2. Moderate risk. ■ 3. High risk. ■ 4. Extremely high risk.

2. A client is at moderate risk when the ANC is less than 1,000. The ANC decreases proportionately to the increased risk for infection. The client is at normal risk for infection if the ANC is 1,500 or greater. The client is at high risk for infection if the ANC is less than 500. An ANC of 100 or less is life-threatening.

128. The nurse interprets which of the following as an early sign of acute respiratory distress syndrome (ARDS) in a client at risk? ■ 1. Elevated carbon dioxide level. ■ 2. Hypoxia not responsive to oxygen therapy. ■ 3. Metabolic acidosis. ■ 4. Severe, unexplained electrolyte imbalance.

2. A hallmark of early ARDS is refractory hypoxemia. The client's PaO2 level continues to fall, despite higher concentrations of administered oxygen. Elevated carbon dioxide and metabolic acidosis occur late in the disorder. Severe electrolyte imbalances are not indicators of ARDS.

30. A client who had received 25 mL of packed red blood cells (PRBCs) has low back pain and pruritus. After stopping the infusion, the nurse should take what action next? ■ 1. Administer prescribed antihistamine and aspirin. ■ 2. Collect blood and urine samples and send to the lab. ■ 3. Administer prescribed diuretics. ■ 4. Administer prescribed vasopressors

2. ABO- and Rh-incompatible blood causes an antigen-antibody reaction that produces hemolysis or agglutination of red blood cells (RBCs). At the fi rst indication of any sign/symptom of reaction, the blood transfusion is stopped. Blood and urine samples are obtained from the client and sent to the lab along with the remaining untransfused blood. Hemoglobin in the urine and blood samples taken at the time of the reaction provides evidence of a hemolytic blood transfusion reaction. Antihistamine, aspirin, diuretics, and vasopressors may be administered with different types of transfusion reactions.

30. A client has driven himself to the emergency department. He is 50 years old, has a history of hypertension, and informs the nurse that his father died from a heart attack at age 60. The client is presently complaining of indigestion. The nurse connects him to an electrocardiogram monitor and begins administering oxygen at 2 L/minute per nasal cannula. The nurse's next action would be to: ■ 1. Call for the physician. ■ 2. Start an I.V. line. ■ 3. Obtain a portable chest radiograph. ■ 4. Draw blood for laboratory studies.

2. Advanced cardiac life support recommends that at least one or two I.V. lines be inserted in one or both of the antecubital spaces. Calling the physician, obtaining a portable chest radiograph, and drawing blood for the laboratory are important but secondary to starting the I.V. line

8. Which of the following explains the infl uence of aging on the development of peripheral vascular disease? ■ 1. Decreased resistance. ■ 2. Increased resistance. ■ 3. Decreased viscosity. ■ 4. Increased viscosity.

2. As people age, the accumulation of collagen in the intima of the blood vessels results in the vessels' becoming stiff and less fl exible. Consequently, there is an increased resistance within the aging adult's circulatory system.

107. During cardiopulmonary resuscitation (CPR), the xiphoid process at the lower end of the sternum should not be compressed when performing cardiac compressions. Which of the following organs would be most likely at risk for laceration by forceful compressions over the xiphoid process? ■ 1. Lung. ■ 2. Liver. ■ 3. Stomach. ■ 4. Diaphragm.

2. Because of its location near the xiphoid process, the liver is the organ most easily damaged from pressure exerted over the xiphoid process during CPR. The pressure on the victim's chest wall should be suffi cient to compress the heart but not so great as to damage internal organs. Injury may result, however, even when CPR is performed properly.

34. A client with microcytic anemia is having trouble selecting food from the hospital menu. Which food is best for the nurse to suggest for satisfying the client's nutritional needs? ■ 1. Egg yolks. ■ 2. Brown rice. ■ 3. Vegetables. ■ 4. Tea.

2. Brown rice is a source of iron from plant sources (nonheme iron). Other sources of nonheme iron are whole-grain cereals and breads, dark green vegetables, legumes, nuts, dried fruits (apricots, raisins, dates), oatmeal, and sweet potatoes. Egg yolks have iron but it is not as well absorbed as iron from other sources. Vegetables are a good source of vitamins that may facilitate iron absorption. Tea contains tannin, which combines with nonheme iron, preventing its absorption

67. The nurse reviews an arterial blood gas report for a client with chronic obstructive pulmonary disease (COPD). pH 7.35; PC02 62; PO2 70; HCO3 34 The nurse should: ■ 1. Apply a 100% non-rebreather mask. ■ 2. Assess the vital signs. ■ 3. Reposition the client. ■ 4. Prepare for intubation

2. Clients with chronic COPD have CO2 retention and the respiratory drive is stimulated when the PO2 decreases. The heart rate, respiratory rate, and blood pressure should be evaluated to determine if the client is hemodynamically stable. Symptoms, such as dyspnea, should also be assessed. Oxygen supplementation, if indicated, should be titrated upward in small increments. There is no indication that the client is experiencing respiratory distress requiring intubation

53. The client with tuberculosis is to be discharged home with community health nursing follow-up. Of the following nursing interventions, which should have the highest priority? ■ 1. Offering the client emotional support. ■ 2. Teaching the client about the disease and its treatment. ■ 3. Coordinating various agency services. ■ 4. Assessing the client's environment for sanitation.

2. Ensuring that the client is well educated about tuberculosis is the highest priority. Education of the client and family is essential to help the client understand the need for completing the prescribed drug therapy to cure the disease. Offering the client emotional support, coordinating various agency services, and assessing the environment may be part of the care for the client with tuberculosis; however, these interventions are of less importance than education about the disease process and its treatment

76. When teaching a client with chronic obstructive pulmonary disease to conserve energy, the nurse should teach the client to lift objects: ■ 1. While inhaling through an open mouth. ■ 2. While exhaling through pursed lips. ■ 3. After exhaling but before inhaling. ■ 4. While taking a deep breath and holding it.

2. Exhaling requires less energy than inhaling. Therefore, lifting while exhaling saves energy and reduces perceived dyspnea. Pursing the lips prolongs exhalation and provides the client with more control over breathing. Lifting after exhaling but before inhaling is similar to lifting with the breath held. This should not be recommended because it is similar to the Valsalva maneuver, which can stimulate cardiac arrhythmias

105. The client is a survivor of non-Hodgkin's lymphoma. Which of the following statements indicates the client needs additional information? ■ 1. "Regular screening is very important for me." ■ 2. "The survivor rate is directly proportional to the incidence of second malignancy." ■ 3. "The survivor rate is indirectly proportional to the incidence of second malignancy." ■ 4. "It is important for survivors to know the stage of the disease and their current treatment plan."

2. It is incorrect that the survivor rate is directly proportional to the incidence of second malignancy. The survivor rate is indirectly proportional to the incidence of second malignancy, and regular screening is very important to detect a second malignancy, especially acute myeloid leukemia or myelodysplastic syndrome. Survivors should know the stage of the disease and their current treatment plan so that they can remain active participants in their health care

14. The nurse is assessing the lower extremities of the client with peripheral vascular disease (PVD). During the assessment, the nurse should expect to fi nd which of the following clinical manifestations of PVD? ■ 1. Hairy legs. ■ 2. Mottled skin. ■ 3. Pink, cool skin. ■ 4. Warm, moist skin.

2. Reduction of blood fl ow to a specifi c area results in decreased oxygen and nutrients. As a result, the skin may appear mottled. Loss of hair and cool, dry skin are other signs that the nurse may observe in a client with PVD of the lower extremities.

81. A client weighs 300 lb (136 kg) and has a history of deep vein thrombosis and thrombophlebitis. When reviewing a teaching plan with this client, the nurse knows that the client has understood the nurse's instructions when he states he will: ■ 1. Avoid exercise. ■ 2. Lose weight. ■ 3. Perform leg lifts every 4 hours. ■ 4. Wear support hose, using rubber bands to hold the stockings up.

2. The client is at risk for development of varicose veins. Therefore, prevention is key in the treatment plan. Maintaining ideal body weight is the goal. In order to achieve this, the client should consume a balanced diet and participate in a regular exercise program. Depending on the individual, leg lifts may or may not be an appropriate activity. Performing leg lifts provides muscular activity and should be done more often than every 4 hours. Wearing support hose is helpful. However, the client should not use rubber bands to hold the stockings up.

54. The nurse is teaching a client with a history of acquired thrombocytopenia about how to prevent and control hemorrhage. Which statement indicates that the client needs further instruction? ■ 1. "I can apply direct pressure over small cuts for at least 5 to 10 minutes to stop a venous bleed." ■ 2. "I can count the number of tissues saturated to detect blood loss during a nosebleed." ■ 3. "I can take hormones to decrease blood loss during menses." ■ 4. "I can count the number of sanitary napkins to detect excess blood loss during menses."

2. The client needs further teaching if she thinks that the number of tissues saturated represents all of the blood lost during a nosebleed. During a nosebleed, a signifi cant amount of blood can be swallowed and go undetected. It is important that clients with severe thrombocytopenia do not take a nosebleed lightly. Clients with thrombocytopenia can apply pressure for 5 to 10 minutes over a small, superfi cial cut. Clients with thrombocytopenia can take hormones to suppress menses and control menstrual blood loss. Clients can also count the number of saturated sanitary napkins to approximate blood loss during menses. Some authorities estimate that a completely soaked sanitary napkin holds 50 mL.

74. A client's arterial blood gas values are as follows: pH, 7.31; PaO2, 80 mm Hg; PaCO2, 65 mm Hg; HCO3 −, 36 mEq/L. The nurse should assess the client for? ■ 1. Cyanosis. ■ 2. Flushed skin. ■ 3. Irritability. ■ 4. Anxiety

2. The high PaCO2 level causes fl ushing due to vasodilation. The client also becomes drowsy and lethargic because carbon dioxide has a depressant effect on the central nervous system. Cyanosis is a sign of hypoxia. Irritability and anxiety are not common with a PaCO2 level of 65 mm Hg but are associated with hypoxia.

93. The nurse should teach the client with asthma that which of the following is one of the most common precipitating factors of an acute asthma attack? ■ 1. Occupational exposure to toxins. ■ 2. Viral respiratory infections. ■ 3. Exposure to cigarette smoke. ■ 4. Exercising in cold temperatures.

2. The most common precipitator of asthma attacks is viral respiratory infection. Clients with asthma should avoid people who have the fl u or a cold and should get yearly fl u vaccinations. Environmental exposure to toxins or heavy particulate matter can trigger asthma attacks; however, far fewer asthmatics are exposed to such toxins than are exposed to viruses. Cigarette smoke can also trigger asthma attacks, but to a lesser extent than viral respiratory infections. Some asthmatic attacks are triggered by exercising in cold weather.

27. The nurse is teaching a client and his family about the client's new diagnosis of hemochromatosis. Which of the following details should the nurse include? ■ 1. Hemochromatosis is an autoimmune disorder that affects the HFE gene. ■ 2. Individuals who are heterozygous for hemochromatosis rarely develop the disease. ■ 3. Individuals who are homozygous for hemochromatosis are carriers of hemochromatosis. ■ 4. Men are at greater risk for hemochromatosis

2. The nurse should teach the client and family that individuals who are heterozygous for hemochromatosis rarely develop the disease. The nurse should teach that men and women are equally at risk for hemochromatosis, but men are diagnosed earlier because women do not usually have manifestations until menopause. Hemochromatosis is the most common genetic disorder in the United States. Individuals who are homozygous for hemochromatosis received a defective gene from each parent. Those with homozygous genes may develop the disease.

15. Which position would most help to decrease a client's discomfort when the client's spouse injects vitamin B12 using the ventrogluteal site? ■ 1. Lying on the side with legs extended. ■ 2. Lying on the abdomen with toes pointed inward. ■ 3. Leaning over the edge of a low table with hips fl exed. ■ 4. Standing upright with the feet one shoulderwidth apart.

2. To promote comfort when injecting at the ventrogluteal site, the position of choice is with the client lying on the abdomen with toes pointed inward. This positioning promotes muscle relaxation, which decreases the discomfort of making an injection into a tense muscle. Lying on the side with legs extended will not provide the greatest muscle relaxation. Leaning over the edge of a table with the hips fl exed and standing upright with the feet apart will increase muscular tension.

19. A client is to have a Schilling test? The nurse should: ■ 1. Administer methylcellulose (Citrucel). ■ 2. Start a 24- to 48-hour urine specimen collection. ■ 3. Maintain nothing-by-mouth (NPO) status. ■ 4. Start a 72-hour stool specimen collection.

2. Urinary vitamin B12 levels are measured after the ingestion of radioactive vitamin B12. A 24- to 48-hour urine specimen is collected after administration of an oral dose of radioactively tagged vitamin B12 and an injection of nonradioactive vitamin B12. In a healthy state of absorption, excess vitamin B12 is excreted in the urine; in a malabsorptive state or when the intrinsic factor is missing, vitamin B12 is excreted in the feces. Citrucel is a bulk-forming agent. Laxatives interfere with the absorption of vitamin B12. The client is NPO 8 to 12 hours before the test but is not NPO during the test. A stool collection is not a part of the Schilling test. If stool contaminates the urine collection, the results will be altered.

A nurse is planning care for a client with a chest tube attached to a Pleur-Evac drainage system. The nurse includes which interventions in the plan? Select all that apply. r 1 Clamping the chest tube intermittently r 2 Changing the client's position frequently r 3 Maintaining the collection chamber below the client's waist r 4 Adding water to the suction control chamber as it evaporates r 5 Taping the connection between the chest tube and the drainage system

2345 To prevent a tension pneumothorax, the nurse avoids clamping the chest tube, unless specifically prescribed. In many facilities, clamping of the chest tube is contraindicated by agency policy. Changing the client's position frequently is necessary to promote drainage and ventilation. Maintaining the system below waist level is indicated to prevent fluid from reentering the pleural space. Adding water to the suction control chamber is an appropriate nursing action and is done as needed to maintain the full suction level prescribed. Taping the connection between the chest tube and system is also indicated to prevent accidental disconnection

nurse is teaching a client newly diagnosed with chronic stable angina. Which instructions should the nurse incorporate in the teaching session on measures to prevent future angina? SELECT ALL THAT APPLY. 1. Increase isometric arm exercises to build endurance. 2. Wear a face mask when outdoors in cold weather. 3. Take nitroglycerin before a stressful situation even though pain is not present. 4. Perform most exertional activities in the morning. 5. Avoid straining at stool. 6. Eliminate tobacco use.

2356 Blood vessels constrict in response to cold and increase the workload of the heart. Sexual activity and straining at stool increases sympathetic stimulation and cardiac workload. Nitroglycerin produces vasodilation and improves blood flow to the coronary arteries; it can be used prophylactically to prevent angina. Nicotine stimulates catecholamine release, producing vasoconstriction and an increased heart rate. Isometric exercise of the arms can cause exertional angina. Exertional activity increases the heart rate, thus reducing the time the heart is in diastole, when blood flow to the coronary arteries is the greatest. A period of rest should occur between activities and activities should be spaced.

22. A client with peripheral vascular disease returns to the surgical care unit after having femoralpopliteal bypass grafting. Indicate in which order the nurse should conduct assessment of this client. 1. Postoperative pain. 2. Peripheral pulses. 3. Urine output. 4. Incision site.

2431 Because assessment of the presence and quality of the pedal pulses in the affected extremity is essential after surgery to make sure that the bypass graft is functioning, this step should be done fi rst. The nurse should next ensure that the dressing is intact, and then that the client has adequate urine output. Lastly, the nurse should determine the client's level of pain.

A client presents to an emergency department following a motorcycle crash. A nurse assesses the client and notes uncoordinated or paradoxical chest rise and fall as well as multiple bruises across the client's chest and torso, crepitus, and tachypnea. Based on this assessment, the nurse should: 1. assist in the placement of a cervical collar. 2. anticipate the need to intubate the client. 3. provide chest compressions. 4. tape the chest wall.

2The assessment data implies a client with multiple broken ribs and potentially flail chest. In the case of flail chest, more invasive interventions are generally required, including management of the client's airway with intubation. The client would most likely already have a cervical collar on, and this is not the intervention that would address the assessment data. There is no evidence to suggest that chest compressions are warranted. Taping the chest wall is an intervention for broken ribs that has proven to not be as effective as once believed.

A nurse observes for early manifestations of acute respiratory distress syndrome (ARDS) in a client being treated for smoke inhalation. Which signs indicates the possible onset of ARDS in this client? 1. Cough with blood-tinged sputum and respiratory alkalosis 2. Decrease in both white and red blood cell counts 3. Diaphoresis and low SaO2 unresponsive to increased oxygen administration 4. Hypertension and elevated PaO2

3 ARDS is manifested and similar to an extreme state of respiratory distress that would include diaphoresis, tachypnea, and use of accessory muscles. Because of damage and alterations in lung tissue, the client would not be able to increase his or her oxygenation despite an increase in the flow or amount of oxygen. Blood pressure and acid-base imbalances vary depending on the stage of ARDS.

A client was admitted to the hospital 24 hours ago after sustaining blunt chest trauma. The nurse monitors for which earliest clinical manifestation of acute respiratory distress syndrome (ARDS)? 1 Cyanosis and pallor 2 Diffuse crackles and rhonchi on chest auscultation 3 Increase in respiratory rate from 18 to 30 breaths per minute 4 Haziness or "white-out" appearance of lungs on chest radiograph

3 Acute respiratory distress syndrome usually develops within 24 to 48 hours after an initiating event, such as chest trauma. In most cases tachypnea and dyspnea are the earliest clinical manifestations as the body compensates for mild hypoxemia through hyperventilation. Cyanosis and pallor are late findings and are the result of severe hypoxemia. Breath sounds in the early stages of ARDS are usually clear but then progress to diffuse crackles and rhonchi as pulmonary edema occurs. Chest radiographic findings may be normal during the early stages but will show diffuse haziness or "white-out" appearance in the later stages.

A primary care provider prescribes lisinopril (Zestril®, Prinivil®) to treat a client with hypertension. The client returns to the clinic for a follow-up appointment. A nurse should evaluate the client for adverse effects by asking the client if he or she is experiencing: 1. muscle weakness. 2. bleeding gums. 3. persistent cough. 4. petechiae

3 Coughing is a common adverse effect of angiotensin-converting enzyme (ACE) inhibitors and warrants discontinuing the medication. The cough occurs from the action of ACE inhibitors in inhibition of kinase II and accumulation of bradykinin. Muscle weakness can be associated with statin therapy and warrants discontinuing the medication. Bleeding gums and petechiae are associated with bleeding. Petechiae are signs of low platelet counts

A nurse is caring for a client in an emergency department who has five fractured ribs from blunt chest trauma. The client is rating pain at 9 out of 10 on a 0 to 10 numeric scale. For which pain management modality should the nurse advocate? 1. NSAIDs 2. Oral analgesics (narcotic + acetaminophen) 3. Regional/local analgesia (epidural or intercostal injection) 4. Intravenous (IV) bolus meperidine (Demerol®)

3 Epidural analgesics and intercostal nerve blocks are the most optimal modality for blunt chest trauma because they directly target the injury site. Oral analgesics generally are not adequate to manage the pain associated with rib fractures. Meperidine is not the ideal narcotic for managing this type of pain because of its multiple adverse side effects.

A client diagnosed with class II heart failure according to the New York Heart Association Functional Classification has been taught about the initial treatment plan for this disease. A nurse determines that the client needs additional teaching if the client states that the treatment plan includes: 1. diuretics. 2. a low-sodium diet. 3. home oxygen therapy. 4. angiotensin-converting enzyme (ACE) inhibitors.

3 In class II heart failure, normal physical activity results in fatigue, dyspnea, palpitations, or anginal pain. The symptoms are absent at rest. Home oxygen therapy is unnecessary unless there are other comorbid conditions. Diuretics mobilize edematous fluid, act on the kidneys to promote excretion of sodium and water, and reduce preload and pulmonary venous pressure. Dietary restriction of sodium aids in reducing edema. ACE inhibitors block the conversion of angiotensin I to the vasoconstrictor angiotensin II, prevent the degradation of bradykinin and other vasodilatory prostaglandins, and increase plasma renin levels and reduce aldosterone levels. The net result is systemic vasodilation, reduced systemic vascular resistance, and improved cardiac output.

A nurse teaches a 55-year-old strict vegetarian that, to decrease the risk of developing megaloblastic anemia, the client should: 1. undergo a Schilling test. 2. increase intake of foods high in iron. 3. supplement the diet with vitamin B12. 4. have a monthly hemoglobin level drawn.

3 Megaloblastic anemia is caused by deficiency of vitamin B12 or folic acid. A vegetarian can prevent a deficiency with oral vitamin supplements or fortified soy milk. The U.S. Department of Agriculture's Dietary Guidelines for Americans 2005 also recommend that people over age 50, whether or not they are vegetarian, consume vitamin B12 in its crystalline form (i.e., fortified foods or supplements). The Schilling test is used to diagnose vitamin B12 deficiency. Consuming foods high in iron will prevent iron-deficiency anemia. Monthly laboratory work is unnecessary and costly.

nurse is performing a respiratory assessment on a client being treated for an asthma attack. The nurse determines that the client's respiratory status is worsening if which of the following occurs? 1 Loud wheezing 2 Wheezing on expiration 3 Noticeably diminished breath sounds 4 Wheezing during inspiration and expiration

3 Noticeably diminished breath sounds are an indication of severe obstruction and impending respiratory failure. Wheezing is not a reliable manifestation to determine the severity of an asthma attack. Clients with minor attacks may experience loud wheezes, whereas others with severe attacks may not wheeze. The client with severe asthma attacks may have no audible wheezing because of the decrease of airflow. For wheezing to occur, the client must be able to move sufficient air to produce breath sounds. Wheezing usually occurs first on expiration. As the asthma attack progresses, the client may wheeze during both inspiration and expiration.

A client learning about chronic obstructive pulmonary disease self-care at a community health class, asks a nurse why the participants are being taught about the "lip-breathing." The nurse should respond by explaining that pursed-lip breathing can help to: 1. reduce upper airway inflammation. 2. reduce anxiety through humor. 3. strengthen respiratory muscles. 4. increase effectiveness of inhaled medications

3 Pursed-lip breathing increases the strength of respiratory muscles and helps to keep alveoli open. It does not have an affect on upper airway inflammation, provide humor therapy, and is not a part of medication administration.

A client with acute respiratory distress syndrome has a prescription to be placed on a continuous positive airway pressure (CPAP) face mask. The nurse implements which of the following for this procedure to be most effective? 1 Obtains baseline arterial blood gases 2 Obtains baseline pulse oximetry levels 3 Applies the mask to the face with a snug fit 4 Encourages the client to remove the mask frequently for coughing and deep breathing exercises

3 The CPAP face mask must be applied over the nose and mouth with a snug fit, which is necessary to maintain positive pressure in the client's airways. The nurse obtains baseline respiratory assessments and arterial blood gases to evaluate the effectiveness of therapy, but these are not done to increase the effectiveness of the procedure. A disadvantage of the CPAP face mask is that the client must remove it for coughing, eating, or drinking. This removes the benefit of positive pressure in the airway each time it is removed.

A client with chronic obstructive pulmonary disease (COPD) is in the third postoperative day following right-sided thoracotomy. During the day shift, the client has required 10 L oxygen by mask to keep his or her oxygen saturations greater than 88%. Based on this information, which action should be taken by the evening shift nurse? 1.Work to wean oxygen down to 3 L by mask 2. Call respiratory therapy for a nebulizer treatment 3. Check respiratory rate and notify the physician 4. Administer dose of ordered pain medications

3 The night shift nurse should check the client's respiratory rate and report abnormal findings to the physician. Although uncommon, clients with COPD on high flow oxygen can lose their respiratory drive. Working to wean down oxygen by mask below 3 L will cause retention of CO2; oxygen by mask generally should be set at 4 L or greater. Although a nebulizer and pain medications may assist the client, the immediate need is to determine if the high flow oxygen is affecting the client's respiratory drive and to further determine the cause of the low oxygen saturations.

The nurse admits a client who is in sickle cell crisis to the hospital. Which does the nurse prepare as the priority in the management of the client? 1 Pain management 2 Fluid administration 3 Oxygen administration 4 Red blood cell transfusion

3 The priority nursing intervention for a client in sickle cell crisis is to administer supplemental oxygen because the client is hypoxemic, and as a result, the red blood cells change to the sickle shape. In addition, oxygen is the priority because airway and breathing are more important than circulatory needs. The nurse also plans for fluid therapy to promote hydration and reverse the agglutination of sickled cells, opioid analgesics for relief from severe pain, and, blood transfusions to increase the blood's oxygen-carrying capacity.

124. The nurse has placed the intubated client with acute respiratory distress syndrome (ARDS) in prone position for 30 minutes. Which of the following would require the nurse to discontinue prone positioning and return the client to the supine position? Select all that apply. ■ 1. The family is coming in to visit. ■ 2. The client has increased secretions requiring frequent suctioning. ■ 3. The SpO2 and PO2 have decreased. ■ 4. The client is tachycardic with drop in blood pressure. ■ 5. The face has increased skin breakdown and edema.

3, 4, 5. The prone position is used to improve oxygenation, ventilation, and perfusion. The importance of placing clients with ARDS in prone positioning should be explained to the family. The positioning allows for mobilization of secretions and the nurse can provide suctioning. Clinical judgment must be used to determine the length of time in the prone position. If the client's hemodynamic status, oxygenation, or skin is compromised, the client should be returned to the supine position for evaluation. Facial edema is expected with the prone position, but the skin breakdown is of concern.

111. Which of the following should be readily available at the bedside of a client with a chest tube in place? ■ 1. A tracheostomy tray. ■ 2. Another sterile chest tube. ■ 3. A bottle of sterile water. ■ 4. A spirometer.

3. A bottle of sterile water should be readily available and in view when a client has a chest tube so that the tube can be immediately submersed in the water if the chest tube system becomes disconnected. The chest tube should be reconnected to the water-seal system as soon as a sterile functioning system can be re-established. There is no need for a tracheostomy tray, another chest tube, or a spirometer to be placed at the bedside for emergency use.

39. A health care provider orders 0.5 mg of protamine sulfate for a client who is showing signs of bleeding after receiving a 100-unit dose of heparin. The nurse should expect the effects of the protamine sulfate to be noted in which of the following time frames? ■ 1. 5 minutes. ■ 2. 10 minutes. ■ 3. 20 minutes. ■ 4. 30 minutes.

3. A dose of 0.5 mg of protamine sulfate reverses a 100-unit dose of heparin within 20 minutes. The nurse should administer protamine sulfate by I.V. push slowly to avoid adverse effects, such as hypotension, dyspnea, bradycardia, and anaphylaxis.

60. When receiving a client from the postanesthesia care unit after a splenectomy, which should the nurse assess after obtaining vital signs? ■ 1. Nasogastric drainage. ■ 2. Urinary catheter. ■ 3. Dressing. ■ 4. Need for pain medication.

3. After a splenectomy, the client is at high risk for hypovolemia and hemorrhage. The dressing should be checked often; if drainage is present, a circle should be drawn around the drainage and the time noted to help determine how fast bleeding is occurring. The nasogastric tube should be connected, but this can wait until the dressing has been checked. A urinary catheter is not needed. The last pain medication administration and the client's current pain level should be communicated in the exchange report. Checking for hemorrhage is a greater priority than assessing pain level.

60. A client with Raynaud's phenomenon is prescribed diltiazem (Cardizem). An expected outcome is: ■ 1. Decreased heart rate. ■ 2. Conversion to normal sinus rhythm. ■ 3. Reduced episodes of fi nger numbness. ■ 4. Increased SpO2.

3. Calcium channel blockers are fi rst-line drug therapy for the treatment of vasospasms with Raynaud's phenomenon when other therapies are ineffective. Cardizem relaxes smooth muscles and improves peripheral perfusion, therefore reducing fi nger numbness. Cardizem decreases heart rate and is used to treat atrial fi brillation, but these are not associated with Raynaud's. When vasospasms are prevented, an accurate SpO2 can be measured in the affected extremity, however SpO2 is a measurement of systemic oxygenation not infl uenced by Cardizem.

64. Which of the following is contraindicated for a client diagnosed with disseminated intravascular coagulation (DIC)? ■ 1. Treating the underlying cause. ■ 2. Administering heparin. ■ 3. Administering warfarin sodium (Coumadin). ■ 4. Replacing depleted blood products

3. DIC has not been found to respond to oral anticoagulants such as warfarin sodium (Coumadin). Treatments for DIC are controversial but include treating the underlying cause, administering heparin, and replacing depleted blood products.

102. A client is undergoing a bone marrow aspiration and biopsy. What is the best way for the nurse to help the client handle her stress? ■ 1. Allow the client's family to stay with her as long as possible. ■ 2. Stay with the client and hold her hand without speaking. ■ 3. Encourage the client to take slow, deep breaths to relax. ■ 4. Allow the client time to express her feelings.

3. Encouraging the client to take slow, deep breaths during uncomfortable parts of procedures is the best method of decreasing the stress response of tightening and tensing the muscles. Slow, deep breathing affects the level of carbon dioxide in the brain to increase the client's sense of well-being. Allowing the client's family to stay with her may be appropriate if the family has a calming effect on the client. Silence can be therapeutic, but when the client is faced with a potentially life-threatening diagnosis and a new, invasive procedure, she really needs words in addition to touch unless another health care provider is talking to her. Expressing feelings is important, but the client will have to hold still for the procedure.

7. The nurse should instruct the client to eat which of the following foods to obtain the best supply of vitamin B12? ■ 1. Whole grains. ■ 2. Green leafy vegetables. ■ 3. Meats and dairy products. ■ 4. Broccoli and brussels sprouts.

3. Good sources of vitamin B12 include meats and dairy products. Whole grains are a good source of thiamine. Green, leafy vegetables are good sources of niacin, folate, and carotenoids (precursors of vitamin A). Broccoli and brussels sprouts are good sources of ascorbic acid (vitamin C).

36. The client who experiences angina has been told to follow a low-cholesterol diet. Which of the following meals should the nurse tell the client would be best on her low-cholesterol diet? ■ 1. Hamburger, salad, and milkshake. ■ 2. Baked liver, green beans, and coffee. ■ 3. Spaghetti with tomato sauce, salad, and coffee. ■ 4. Fried chicken, green beans, and skim milk

3. Pasta, tomato sauce, salad, and coffee would be the best selection for the client following a low-cholesterol diet. Hamburgers, milkshakes, liver, and fried foods tend to be high in cholesterol.

104. A client with advanced Hodgkin's disease is admitted to hospice because death is imminent. The goal of nursing care is to help relieve the client's: ■ 1. Fear of pain. ■ 2. Fear of further therapy. ■ 3. Feelings of isolation. ■ 4. Feelings of social inadequacy.

3. Terminally ill clients most often describe feelings of isolation because they tend to be ignored; they are often left out of conversations (especially those dealing with the future); and they sense the attitudes of discomfort that many people feel in their presence. Helpful nursing measures include taking the time to be with the client; offering opportunities to talk about feelings; and answering questions honestly.

24. When comparing the hematocrit levels of a postoperative client, the nurse notes that the hematocrit decreased from 36% to 34% on the third day even though the RBC count and hemoglobin value remained stable at 4.5 million/μL and 11.9 g/dL, respectively. The nurse should next: ■ 1. Check the dressing and drains for frank bleeding. ■ 2. Call the physician. ■ 3. Continue to monitor vital signs. ■ 4. Start oxygen at 2 L/minute per nasal cannula.

3. The nurse should continue to monitor the client because this value refl ects a normal physiologic response. The physician does not need to be called, and oxygen does not need to be started based on these laboratory fi ndings. Immediately after surgery, the client's hematocrit refl ects a falsely high value related to the body's compensatory response to the stress of sudden loss of fl uids and blood. Activation of the intrinsic pathway and the renin-angiotensin cycle via antidiuretic hormone produces vasoconstriction and retention of fl uid for the fi rst 1 to 2 days postoperatively. By the second to third day, this response decreases and the client's hematocrit level is more refl ective of the amount of RBCs in the plasma. Fresh bleeding is a less likely occurrence on the third postoperative day but is not impossible; however, the nurse should have expected to see a decrease in the RBC count and hemoglobin value accompanying the hematocrit.

81. The nurse administers theophylline (Theo- Dur) to a client. To evaluate the effectiveness of this medication, which of the following drug actions should the nurse anticipate? ■ 1. Suppression of the client's respiratory infection. ■ 2. Decrease in bronchial secretions. ■ 3. Relaxation of bronchial smooth muscle. ■ 4. Thinning of tenacious, purulent sputum.

3. Theophylline (Theo-Dur) is a bronchodilator that is administered to relax airways and decrease dyspnea. Theophylline is not used to treat infections and does not decrease or thin secretions.

103. While assessing a thoracotomy incisional area from which a chest tube exits, the nurse feels a crackling sensation under the fi ngertips along the entire incision. Which of the following should be the nurse's fi rst action? ■ 1. Lower the head of the bed and call the physician. ■ 2. Prepare an aspiration tray. ■ 3. Mark the area with a skin pencil at the outer periphery of the crackling. ■ 4. Turn off the suction of the chest drainage system.

3. This crackling sensation is subcutaneous emphysema. Subcutaneous emphysema is not an unusual fi nding, and it is not dangerous if confi ned. But progression can be serious, especially if the neck is involved; a tracheotomy may be needed. If emphysema progresses noticeably in 1 hour, the physician should be notifi ed. Lowering the head of the bed will not arrest the progress or provide any further information. A tracheotomy tray would be useful if subcutaneous emphysema progresses to the neck. Subcutaneous emphysema may progress if the chest drainage system does not adequately remove air and fl uid; therefore, the system should not be turned off.

A child is admitted to the hospital with a suspected diagnosis of idiopathic thrombocytopenic purpura (ITP), and diagnostic studies are performed. Which of the following diagnostic results are indicative of this disorder? 1 An elevated platelet count 2 Elevated hemoglobin and hematocrit levels 3 A bone marrow examination showing an increased number of megakaryocytes 4 A bone marrow examination indicating an increased number of immature white blood cells

33 The laboratory manifestations of ITP include the presence of a low platelet count of usually less than 20,000 cells/mm3. Thrombocytopenia is the only laboratory abnormality expected with ITP. If there has been significant blood loss, there is evidence of anemia in the blood cell count. If a bone marrow examination is performed, the results with ITP show a normal or increased number of megakaryocytes, which are the precursors of platelets. Option 4 indicates the bone marrow result that would be found in a child with leukemia.

A client has been defibrillated three times using an automatic external defibrillator (AED). The nurse observes that the attempts to convert the ventricular fibrillation (VF) were unsuccessful. Based on an evaluation of the situation, the nurse determines that which action would be best? 1 Terminating the resuscitation effort 2 Preparing for the administration of sodium bicarbonate intravenously 3 Performing cardiopulmonary resuscitation (CPR) for 5 minutes, then defibrillating three more times at 400 joules 4 Performing cardiopulmonary resuscitation (CPR) for 1 minute, then defibrillating up to three more times at 360 joules

4 After three unsuccessful defibrillation attempts using an AED, CPR should be done for 1 minute, followed by three more shocks, each delivered at 360 joules. There is no information in the question to indicate that life support should be terminated. Sodium bicarbonate may be prescribed but is not the best action. Giving CPR for 5 minutes may not help oxygenation to the brain and myocardium and is not the best action. It would be best to administer CPR for 1 minute and then resume attempts to convert the rhythm to a viable one.

Chemical cardioversion is prescribed for the client with atrial fibrillation. The nurse who is assisting in preparing the client would expect that which medication specific for chemical cardioversion will be needed? 1 Nitroglycerin 2 Nifedipine (Procardia) 3 Lidocaine (Xylocaine) 4 Amiodarone (Cordarone)

4 Amiodarone is an antidysrhythmic that is useful in restoring normal sinus rhythm for the client experiencing atrial fibrillation. Both nitroglycerin and nifedipine are vasodilators. Lidocaine is used for control of ventricular dysrhythmias.

On the first postoperative day following right-sided thoracotomy, a nurse is assisting a client with arm and shoulder exercises. The client reports pain with the exercises and wants to know why they must be performed. The nurse should explain that the exercises: 1. promote respiratory function. 2. increase blood flow back to the heart and venous system. 3. improve muscle mass to compensate for muscle removed during the procedure. 4. prevent stiffening and loss of function.

4 Because of the location of the incision, disuse can cause contractures and loss of muscle tone. The exercises help to preserve function of the arm and shoulder. Activity will promote respiratory function and improve venous return, but these are not the reasons for the exercises. Although the girdle muscles are cut, they are generally not removed.

A client with tuberculosis (TB) is preparing for discharge from the hospital, and the nurse provides instructions to the client about home care. Which client statement indicates that further instructions are necessary? 1 "I need to place used tissues in a plastic bag when I am home." 2 "I need to eat foods that are high in iron, protein, and vitamin C." 3 "It is not necessary to maintain respiratory isolation when I am at home." 4 "If I miss a dose of medication because of nausea, I just skip that dose and then resume my regular schedule

4 Because of the resistant strains of TB, the nurse must emphasize that noncompliance regarding medication could lead to an infection that is difficult to treat and that may cause total drug resistance. Clients may prevent nausea related to the medications by taking the daily dose at bedtime, and antinausea medications may also prevent this symptom. Medication doses should not be skipped. Options 1, 2, and 3 are correct statements.

A client who has had an abdominal aortic aneurysm repair is 1 day postoperative. The nurse performs an abdominal assessment and notes the absence of bowel sounds. The nurse should: 1 Feed the client. 2 Call the physician immediately. 3 Remove the nasogastric (NG) tube. 4 Document the finding and continue to assess for bowel sounds.

4 Bowel sounds may be absent for 3 to 4 days postoperative because of bowel manipulation during surgery. The nurse should document the finding and continue to monitor the client. The NG tube should stay in place if present, and the client is kept NPO until after the onset of bowel sounds. Additionally, the nurse does not remove the tube without a prescription to do so. There is no need to call the physician immediately at this time.

A client remains in atrial fibrillation with rapid ventricular response despite pharmacological intervention. Synchronous cardioversion is scheduled to convert the rapid rhythm. The nurse plans for implementation of which important action to ensure safety and prevent complications of this procedure? 1 Cardiovert the client at 360 joules. 2 Sedate the client before cardioversion. 3 Ensure that emergency equipment is available. 4 Ensure that the defibrillator is set on the synchronous mode.

4 Cardioversion is similar to defibrillation with two major exceptions: (1) the countershock is synchronized to occur during ventricular depolarization (QRS complex), and (2) less energy is used for the countershock. The rationale for delivering the shock during the QRS complex is to prevent the shock from being delivered during repolarization (T wave), often termed the "vulnerable period." If the shock is delivered during this period, the resulting complication is ventricular fibrillation. It is crucial that the defibrillator is set on the "synchronous" mode for a successful cardioversion. Cardioversion usually begins with 50 to 100 joules. Options 2 and 3 will not prevent complications

Which does the nurse assess for in a client who has pernicious anemia? 1 Constipation 2 Shortness of breath 3 Dusky lips and gums 4 Smooth, sore, red tongue

4 Classic clinical indicators of pernicious anemia include weakness, mild diarrhea, and a smooth, sore red tongue. The client may also have neurological findings, such as paresthesias, confusion, and difficulty with balance. Pernicious anemia does not affect tissue oxygenation, so the mucous membranes do not become dusky, and the client does not exhibit shortness of breath (options 2 and 3). Constipation is not a common finding with pernicious anemia.

A nurse hears the alarm sound on the telemetry monitor, looks at the monitor, and notes that a client is in ventricular tachycardia. The nurse rushes to the client's room. Upon reaching the client's bedside, the nurse should take which action first? 1 Call a code. 2 Prepare for cardioversion. 3 Prepare to defibrillate the client. 4 Check the client's level of consciousness.

4 Determining unresponsiveness is the first assessment action to take. When a client is in ventricular tachycardia, there is a significant decrease in cardiac output. However, assessing for unresponsiveness helps to determine whether the client is affected by the decreased cardiac output. If the client is unconscious, cardiopulmonary resuscitation is initiated Give O2 and antiarrythmia

During a code, a physician is about to defibrillate a client in ventricular fibrillation and says in a loud voice "CLEAR!" Which of the following should the nurse immediately perform? 1 Shut off the mechanical ventilator. 2 Shut off the intravenous infusion going into the client's arm. 3 Place the conductive gel pads for defibrillation on the client's chest. 4 Step away from the bed and make sure that all others have done the same

4 For the safety of all personnel, when the defibrillator paddles are being discharged, all personnel must stand back and be clear of all contact with the client or the client's bed. It is the primary responsibility of the person defibrillating to communicate the "clear" message loudly enough for all to hear and ensure their compliance. All personnel must immediately comply with this command. A ventilator is not in use during a code; rather an Ambu (resuscitation) bag is used. Shutting off the intravenous infusion has no useful purpose. The gel pads should have been placed on the client's chest before the defibrillator paddles were applied. Stepping back from the bed prevents the nurse or others from being defibrillated along with the client.

client taking medication for treatment of essential hypertension has a serum potassium level of 3.2 mEq/L. A nurse is reviewing the list of medications being taken by the client. Which medication on the list should the nurse conclude to be the causative factor for this serum potassium level? 1. Spironolactone (Aldactone®) 2. Potassium chloride (K-Dur®) 3. Enalapril (Vasotec®) 4. Hydrochlorothiazide (Esidrix®, HydroDIURIL®)

4 Hydrochlorothiazide is a thiazide diuretic that blocks sodium and water reabsorption in the distal tubule of the kidney and promotes potassium excretion, putting the client at risk for hypokalemia. Spironolactone acts by inhibiting sodium reabsorption in exchange for potassium (potassium-sparing diuretic). Potassium chloride is a potassium supplement. Enalapril is an angiotensin-converting enzyme (ACE) inhibitor that causes some clients to retain potassium.

A nurse is assisting a client with a chest tube to get out of bed, and the chest tubing accidentally gets caught in the bed rail and disconnects. While trying to reestablish the connection, the Pleur-Evac drainage system falls over and cracks. The nurse takes which immediate action? 1 Calls the physician 2 Clamps the chest tube 3 Applies a petrolatum gauze over the end of the chest tube 4 Immerses the chest tube in a bottle of sterile normal saline

4 If a chest tube accidentally disconnects from the tubing of the drainage apparatus, the nurse should first reestablish an underwater seal to prevent tension pneumothorax and mediastinal shift. This can be accomplished by reconnecting the chest tube, or in this case, immersing the end of the chest tube in a bottle of sterile normal saline or water. The physician should be notified after taking corrective action. If the physician is called first, tension pneumothorax has time to develop. Clamping the chest tube could also cause tension pneumothorax. A petrolatum gauze would be applied to the skin over the chest tube insertion site if the entire chest tube was accidentally removed from the chest

A client has developed oral mucositis as a result of radiation to the head and neck. The nurse should teach the client to incorporate which of the following measures in his or her daily home care routine? 1 Oral hygiene should be performed in the morning and evening. 2 A glass of wine per day will not pose any further harm to the oral cavity. 3 High-protein foods such as peanut butter should be incorporated in the diet. 4 A combination of a weak saline and water solution should be used to rinse the mouth before and after each meal.

4 Oral mucositis (irritation, inflammation, and/or ulceration of the mucosa) also known as stomatitis, commonly occurs in clients receiving radiation to the head and neck. Measures need to be taken to soothe the mucosa as well as provide effective cleansing of the oral cavity. A combination of a weak saline and water solution is an effective cleansing agent. Oral hygiene should be performed more frequently than in the morning and evening. Alcohol would dry and irritate the mucosa. Peanut butter has a thick consistency and will stick to the irritated mucosa

The nurse is performing an admission assessment for a client admitted to the hospital with a diagnosis of Raynaud's disease. The nurse assesses for the symptoms associated with Raynaud's disease by: 1 Checking for a rash on the digits 2 Observing for softening of the nails or nail beds 3 Palpating for a rapid or irregular peripheral pulse 4 Palpating for diminished or absent peripheral pulses

4 Raynaud's disease is vasospasm of the arterioles and arteries of the upper and lower extremities. It produces closure of the small arteries in the distal extremities in response to cold, vibration, or external stimuli. Palpation for diminished or absent peripheral pulses checks for interruption of circulation. Skin changes include hair loss, thinning or tightening of the skin, and delayed healing of cuts or injuries. A rash on the digits is not a characteristic of this disorder. The nails grow slowly, become brittle or deformed, and heal poorly around the nail beds when infected. Although palpation of peripheral pulses is correct, a rapid or irregular pulse would not be noted.

nurse is providing emergency treatment for a client in ventricular tachycardia and is preparing to defibrillate the client. Which nursing action provides for the safest environment during a defibrillation attempt? 1 Ensuring that no lubricant is on the paddles 2 Placing the charged paddles one at a time on the client's chest 3 Holding the client's upper torso stable while the defibrillation is performed 4 Performing a visual and verbal check that all assisting personnel are clear of the client and the client's bed

4 Safety during defibrillation is essential for preventing injury to the client and the personnel assisting with the procedure. The person performing the defibrillation ensures that all personnel are standing clear of the bed by a verbal and visual check of "all clear." For the shock to be effective, some type of conductive medium (e.g., lubricant, gel) must be placed between the paddles and the skin. Both paddles are placed on the client's chest.

The nurse in an ambulatory clinic administers a tuberculin (Mantoux) skin test to a client on a Monday. The nurse tells the client to return to the clinic to have the results read on: 1 Thursday or Friday 2 The following Monday 3 Tuesday or Wednesday 4 Wednesday or Thursday

4 The Mantoux skin test for tuberculosis is read in 48 to 72 hours. Therefore the client should return to the clinic on Wednesday or Thursday

The nurse has provided instructions to a client who is receiving external radiation therapy. Which statement by the client indicates a need for further instructions regarding self-care related to the radiation therapy? 1 "I need to eat a high-protein diet." 2 "I need to avoid exposure to sunlight." 3 "I need to wash my skin with a mild soap and pat it dry." 4 "I need to apply pressure on the irritated area to prevent bleeding."

4 The client receiving external radiation therapy should avoid pressure on the irritated area and wear loose-fitting clothing. Specific physician instructions would be necessary to obtain if an alteration in skin integrity occurred as a result of the radiation therapy. Options 1, 2, and 3 are accurate measures regarding radiation therapy.

A client is suspected of having pulmonary tuberculosis. The nurse assesses the client for which signs and symptoms of tuberculosis? 1 High fever and chest pain 2 Increased appetite, dyspnea, and chills 3 Weight gain, insomnia, and night sweats 4 Low-grade fever, fatigue, and productive cough

4 The client with pulmonary tuberculosis generally has a productive or nonproductive cough, anorexia and weight loss, fatigue, low-grade fever, chills and night sweats, dyspnea, hemoptysis, and chest pain. Breath sounds may reveal crackles.

A nurse admits a client to a hospital and obtains a nursing history. The client tells the nurse that he had an endovascular repair of an abdominal aortic aneurysm found 1 year earlier during a routine screening. The nurse understands that this procedure consists of: 1. excision of the aneurysm and placement of a graft percutaneously. 2. an angioplasty with placement of a stent around the outside of the aorta. 3. placement of a filter within the aneurysm to block clots from becoming emboli. 4. placement of a stent graft inside the aorta that excludes the aneurysm from circulation.

4 The endovascular repair consists of placement of the endovascular stent graft inside the aorta extending above and below the aneurysmal area to seal it off from the circulation. The aneurysm is left in place. Angioplasty (ballooning of plaque from the inside) is not

A client comes into the health clinic 3 years after undergoing a resection of the terminal ileum complaining of weakness, shortness of breath, and a sore tongue. Which client statement indicates a need for intervention and client teaching? A "I have been drinking plenty of fluids." B "I have been gargling with warm salt water for my sore tongue." C "I have 3 to 4 loose stools per day." D "I take a vitamin B12 tablet every day.

4 Vitamin B12 combines with intrinsic factor in the stomach and is then carried to the ileum, where it is absorbed in the bloodstream. In this situation, vitamin B12 cannot be absorbed regardless of the amount of oral intake of sources of vitamin B12 such as animal protein or vitamin B12 tablets. Vitamin B12 needs to be injected every month, because the ileum has been surgically removed. Replacement of fluids and electrolytes is important when the client has continuous multiple loose stools on a daily basis. Warm salt water is used to soothe sore mucous membranes. Crohn's disease and small bowel resection may cause several loose stools a day.

When reviewing the laboratory results of a client with leukemia who is receiving chemotherapy, the registered nurse notes that the neutrophil count is less than 500/mm3. The registered nurse informs the nursing student caring for the client about the results and asks the student to identify the appropriate precautions that need to be instituted. Which intervention identified by the student indicates a need for teaching? 1 Restricting visitors with colds or respiratory infections 2 Removing all live plants, flowers, and stuffed animals in the client's room 3 Placing the client on a low-bacteria diet that excludes raw foods and vegetables 4 Padding the side rails and removing all hazardous and sharp objects from the envt

4 When the neutrophil count is less than 500/mm3, visitors should be screened for the presence of infection, and any visitors or staff with colds or respiratory infections should not be allowed in the client's room. All live plants, flowers, and stuffed animals are removed from the client's room. The client is placed on a low-bacteria diet that excludes raw fruits and vegetables. Padding the side rails and removing all hazardous and sharp objects from the environment would be instituted if the client is at risk for bleeding. This client is at risk for infection

32. When beginning I.V. erythropoietin (Epogen, Procrit) therapy, the nurse should do which of the following? Select all that apply. ■ 1. Checking the hemoglobin levels before administering subsequent doses. ■ 2. Shaking the vial thoroughly to mix the concentrated white, milky solution. ■ 3. Keeping the multidose vial refrigerated between scheduled twice-a-day doses. ■ 4. Administering the medication through the I.V. line without other medications. ■ 5. Adjusting the initial doses according to the client's changes in blood pressure. ■ 6. Educating the client to avoid driving and performing hazardous

4, 5, 6. Erythropoietin is administered to decrease the need for blood transfusions by stimulating RBC production. The medication should be administered through the I.V. line without other medications to avoid a reaction. The hematocrit, a simple measurement of the percent of RBCs in the total blood volume, is used to monitor this therapy. When initiating I.V. erythropoietin therapy, the nurse should monitor the hematocrit level so that it rises no more than four points in any 2-week period. In addition, the initial doses of erythropoietin are adjusted according to the client's changes in blood pressure. The nurse should tell the client to avoid driving and performing hazardous activity during the initial treatment due to possible dizziness and headaches secondary to the adverse effect of hypertension. The hematocrit, not the hemoglobin level, is used for monitoring the effectiveness of therapy. The vial of erythropoietin should not be shaken because it may be biologically inactive. The solution should not be used if it is discolored. The nurse should not reenter the vial once it has been entered; it is a one-time use vial. All remaining erythropoietin should be discarded since it does not contain preservatives.

42. A client with a history of systemic lupus erythematosus was admitted with a severe viral respiratory tract infection and diffuse petechiae. Based on these data, it is most important that the nurse further evaluate the client's recent: ■ 1. Quality and quantity of food intake. ■ 2. Type and amount of fl uid intake. ■ 3. Weakness, fatigue, and ability to get around. ■ 4. Length and amount of menstrual fl ow.

4. A recent viral infection in a female client between the ages of 20 and 30 with a history of systemic lupus erythematosus and an insidious onset of diffuse petechiae are hallmarks of idiopathic thrombocytopenic purpura. It is important to ask whether the client's recent menses have been lengthened or are heavier. Determining her ability to clot can help determine her risk of increased bleeding tendency until a platelet count is drawn. Petechiae are not caused by poor nutrition. Because of poor food and fl uid intake or weakness and fatigue, the client may have gotten bruises from falling or bumping into things, but not petechiae.

141. The nurse should place a client being admitted to the hospital with suspected tuberculosis on what type of isolation? ■ 1. Standard precautions. ■ 2. Contact precautions. ■ 3. Droplet precautions. ■ 4. Airborne precautions.

4. Airborne precautions prevent transmission of infectious agents that remain infectious over long distances when suspended in the air (e.g. mycobacterium tuberculosis, measles, varicella virus [chickenpox], and possibly SARS-CoV). The preferred placement is in an isolation single-client room that is equipped with special air handling and ventilation. A negative pressure room, or an area that exhausts room air directly outside or through HEPA fi lters, should be used if recirculation is unavoidable. Standard precautions combine the major features of Universal Precautions and Body Substance Isolation and are based on the principle that the blood, body fl uids, secretions, and excretions of all clients may contain transmissible infectious agents. Standard precautions include: hand hygiene; use of gloves, gown, mask, eye protection, or face shield, depending on the anticipated exposure; and safe injection practices. Contact precautions are for clients with known or suspected infections or evidence of syndromes that represent an increased risk for contact transmission. Droplet precautions are intended to prevent transmission of pathogens spread through close respiratory or mucous membrane contact with respiratory secretions. Because these pathogens do not remain infectious over long distances in a health care facility, special air handling and ventilation are not required to prevent droplet transmission

85. The nurse is assessing a client with chronic myeloid leukemia (CML). The nurse should assess the client for: ■ 1. Lymphadenopathy. ■ 2. Hyperplasia of the gum. ■ 3. Bone pain from expansion of marrow. ■ 4. Shortness of breath..

4. Although the clinical manifestations of CML vary, clients usually have confusion and shortness of breath related to decreased capillary perfusion to the brain and lungs. Lymphadenopathy is rare in CML. Hyperplasia of the gum and bone pain are clinical manifestations of AML.

1. The nurse is assessing a 48-year-old client with a history of smoking during a routine clinic visit. The client, who exercises regularly, reports having pain in the calf during exercise that disappears at rest. Which of the following fi ndings requires further evaluation? ■ 1. Heart rate 57 bpm. ■ 2. SpO2 of 94% on room air. ■ 3. Blood pressure 134/82. ■ 4. Ankle brachial index of 0.65.

4. An Ankle Brachial Index of 0.65 suggests moderate arterial vascular disease in a client who is experiencing intermittent claudication. Abnormal if 0.9 or lower and 1.40 or higher. .A Doppler ultrasound is indicated for further evaluation. The bradycardic heart rate is acceptable in an athletic client with a normal blood pressure. The SpO2 is acceptable; the client has a smoking history.

135. Which one of the following assessments is most appropriate for determining the correct placement of an endotracheal tube in a mechanically ventilated client? ■ 1. Assessing the client's skin color. ■ 2. Monitoring the respiratory rate. ■ 3. Verifying the amount of cuff infl ation. ■ 4. Auscultating breath sounds bilaterally.

4. Auscultation for bilateral breath sounds is the most appropriate method for determining cuff placement. The nurse should also look for the symmetrical rise and fall of the chest and should note the location of the exit mark on the tube. Assessments of skin color, respiratory rate, and the amount of cuff infl ation cannot validate the placement of the endotracheal tube.

91. Which of the following is an appropriate expected outcome for an adult client with wellcontrolled asthma? ■ 1. Chest X-ray demonstrates minimal hyperinfl ation. ■ 2. Temperature remains lower than 100° F (37.8° C). ■ 3. Arterial blood gas analysis demonstrates a decrease in PaO2. ■ 4. Breath sounds are clear.

4. Between attacks, breath sounds should be clear on auscultation with good air fl ow present throughout lung fi elds. Chest X-rays should be normal. The client should remain afebrile. Arterial blood gases should be normal

47. A client who is taking Bufferin Arthritis Strength caplets develops prolonged bleeding from a superfi cial skin injury on the forearm. The nurse should tell the client to do which of the following fi rst? ■ 1. Place the forearm under a running stream of lukewarm water. ■ 2. Pat the injury with a dry washcloth. ■ 3. Wrap the entire forearm from the wrist to the elbow. ■ 4. Apply an ice pack for 20 minutes

4. Bufferin contains aspirin, which is an anticoagulant; bleeding time can be prolonged. Intermittent use of ice packs to the site may stop the bleeding; ice causes blood vessels to vasoconstrict. Use of lukewarm water, patting the injury, and wrapping the entire forearm do not promote vasoconstriction to stop bleeding.

88. In assessing a client in the early stage of chronic lymphocytic leukemia (CLL), the nurse should determine if the client has: ■ 1. Enlarged, painless lymph nodes. ■ 2. Headache. ■ 3. Hyperplasia of the gums. ■ 4. Unintentional weight loss.

4. Clients with CLL develop unintentional weight loss; fever and drenching night sweats; enlarged, painful lymph nodes, spleen, and liver; decreased reaction to skin sensitivity tests (anergy); and susceptibility to viral infections. Enlarged, painless lymph nodes are a clinical manifestation of Hodgkin's lymphoma. A headache would not be one of the early signs and symptoms expected in CLL because CLL does not cross the blood-brain barrier and would not irritate the meninges. Hyperplasia of the gums is a clinical manifestation of AML

31. Crackles heard on lung auscultation indicate which of the following? ■ 1. Cyanosis. ■ 2. Bronchospasm. ■ 3. Airway narrowing. ■ 4. Fluid-fi lled alveoli.

4. Crackles are auscultated over fl uid-fi lled alveoli. Crackles heard on lung auscultation do not have to be associated with cyanosis. Bronchospasm and airway narrowing generally are associated with wheezing sounds

56. Which of the following family members exposed to tuberculosis would be at highest risk for contracting the disease? ■ 1. 45-year-old mother. ■ 2. 17-year-old daughter. ■ 3. 8-year-old son. ■ 4. 76-year-old grandmother.

4. Elderly persons are believed to be at higher risk for contracting tuberculosis because of decreased immunocompetence. Other high-risk populations in the United States include the urban poor, clients with acquired immunodefi ciency syndrome, and minority groups.

50. The client with idiopathic thrombocytopenic purpura (ITP) asks the nurse why she has to take steroids. Which is the nurse's best response? ■ 1. Steroids destroy the antibodies and prolong the life of platelets. ■ 2. Steroids neutralize the antigens and prolong the life of platelets. ■ 3. Steroids increase phagocytosis and increase the life of platelets. ■ 4. Steroids alter the spleen's recognition of platelets and increase the life of platelets.

4. ITP is treated with steroids to suppress the splenic macrophages from phagocytizing the antibody-coated platelets, which are recognized as foreign bodies, so that the platelets live longer. The steroids also suppress the binding of the autoimmune antibody to the platelet surface. Steroids do not destroy the antibodies on the platelets, neutralize antigens, or increase phagocytosis

32. A client is scheduled to undergo right axillary-to-axillary artery bypass surgery. Which of the following interventions is most important for the nurse to implement in the preoperative period? ■ 1. Assess the temperature in the affected arm. ■ 2. Monitor the radial pulse in the affected arm. ■ 3. Protect the extremity from cold. ■ 4. Avoid using the arm for a venipuncture

4. If surgery is scheduled, the nurse should avoid venipunctures in the affected extremity. The goal should be to prevent unnecessary trauma and possible infection in the affected arm. Disruptions in skin integrity and even minor skin irritations can cause the surgery to be canceled. The nurse can continue to monitor the temperature and radial pulse in the affected arm; however, doing so is not the priority. Keeping the client warm is important but is not the priority at this time

69. Which of the following indicates that the client with chronic obstructive pulmonary disease (COPD) who has been discharged to home understands his care plan? ■ 1. The client promises to do pursed-lip breathing at home. ■ 2. The client states actions to reduce pain. ■ 3. The client says that he will use oxygen via a nasal cannula at 5 L/minute. ■ 4. The client agrees to call the physician if dyspnea on exertion increases

4. Increasing dyspnea on exertion indicates that the client may be experiencing complications of COPD. Therefore, the nurse should notify the physician. Extracting promises from clients is not an outcome criterion. Pain is not a common symptom of COPD. Clients with COPD use low-fl ow oxygen supplementation (1 to 2 L/minute) to avoid suppressing the respiratory drive, which, for these clients, is stimulated by hypoxia.

127. Which of the following interventions would be most likely to prevent the development of acute respiratory distress syndrome (ARDS)? ■ 1. Teaching cigarette smoking cessation. ■ 2. Maintaining adequate serum potassium levels. ■ 3. Monitoring clients for signs of hypercapnia. ■ 4. Replacing fl uids adequately during hypovolemic states

4. One of the major risk factors for development of ARDS is hypovolemic shock. Adequate fl uid replacement is essential to minimize the risk of ARDS in these clients. Teaching smoking cessation does not prevent ARDS. An abnormal serum potassium level and hypercapnia are not risk factors for ARDS.

125. The nurse has calculated a low PaO2/FIO2 (P/F) ratio < 150 for a client with acute respiratory distress syndrome (ARDS). The nurse should place the client in which position to improve oxygenation, ventilation distribution, and drainage of secretions? ■ 1. Supine. ■ 2. Semi-fowlers. ■ 3. Lateral side. ■ 4. Prone.

4. Prone positioning is used to improve oxygenation in clients with acute respiratory distress syndrome (ARDS) who are receiving mechanical ventilation. The positioning allows for recruitment of collapsed alveolar units, improvement in ventilation, reduction in shunting, mobilization of secretions, and improvement in functional reserve capacity (FRC). When the client is supine, side-to-side repositioning should be done every 2 hours with the head of the bed elevated at least 30 degrees.

77. A client with deep vein thrombosis (DVT) has an edematous right lower extremity. The client lies on her right side frequently. Rubor is noted on the lateral aspect of the right ankle. From the data collected, the appropriate nursing diagnosis for this client would be: ■ 1. Activity intolerance related to complaints of pain in lower right extremity. ■ 2. Ineffective health maintenance related to lack of knowledge about DVT. ■ 3. Pain related to edema. ■ 4. Risk for impaired skin integrity.

4. Risk for impaired skin integrity is the primary nursing diagnosis. With rubor or hyperemia, there is increased blood fl ow to the area, raising fi ltration pressure. As a result, capillary permeability is altered, causing damage to capillary walls. The increased permeability, obstruction of lymphatic drainage, elevation of venous pressure, and decrease in plasma protein osmotic force result in edema. The data do not support the nursing diagnoses of Activity intolerance, Ineffective health maintenance, or Pain.

76. The nurse should remind the unlicensed personnel that which of the following is the most important goal in the care of the neutropenic client in isolation? ■ 1. Listening to the client's feelings of concern. ■ 2. Completing the client's care in a nonhurried manner. ■ 3. Completing all of the client's care at one time. ■ 4. Instructing the client to dispose of tissue after blowing the nose.

4. The most common source of infection and microbial colonization in neutropenic clients is their own nonpathogenic normal fl ora. Attention to personal hygiene, such as oral, pulmonary, urinary, and rectal care, is essential. It is important to acknowledge the client's concerns and fears and to provide organized, nonhurried, caring care, but it is more important to teach the client how to prevent an infection that could be life-threatening.

11. A client comes to the health clinic 3 years after undergoing a resection of the terminal ileum and tells the nurse that he has weakness, shortness of breath, and a sore tongue. Which client statement indicates a need for intervention and client teaching? ■ 1. "I have been drinking plenty of fl uids." ■ 2. "I have been gargling with warm salt water for my sore tongue." ■ 3. "I have three to four loose stools per day." ■ 4. "I take a vitamin B12 tablet every day."

4. Vitamin B12 combines with intrinsic factor in the stomach and is then carried to the ileum, where it is absorbed into the bloodstream. In this situation, vitamin B12 cannot be absorbed regardless of the amount of oral intake of sources of vitamin B12, such as animal protein or vitamin B12 tablets. Vitamin B12 needs to be injected every month because the ileum has been surgically removed. Replacement of fl uids and electrolytes is important when the client has continuous multiple loose stools on a daily basis. Warm salt water is used to soothe sore mucous membranes. Crohn's disease and a small-bowel resection may cause several loose stools a day.

33. A client states that she is afraid of receiving vitamin B12 injections because of potential toxic reactions. Which is the nurse's best response to relieve these fears? ■ 1. "Vitamin B12 will cause ringing in the ears before a toxic level is reached." ■ 2. "Vitamin B12 may cause a very mild rash initially." ■ 3. "Vitamin B12 cause mild nausea but nothing toxic." ■ 4. "Vitamin B12 is generally free of toxicity because it is water-soluble

4. Vitamin B12 is a water-soluble vitamin. When water-soluble vitamins are taken in excess of the body's needs, they are fi ltered through the kidneys and excreted. Vitamin B12 is considered to be nontoxic. Adverse reactions that have occurred are believed to be related to impurities or to the preservative in B12 preparations. Ringing in the ears, rash, and nausea are not considered to be related to vitamin B12 administration.

A client seeks medical attention because of pain that develops while walking. An ankle-brachial index (ABI) test is ordered, and the results show that the client has ratios of 1.4 and 1.3 bilaterally. Based on these results, a nurse determines that the client: 1. has severe peripheral arterial disease. 2. would benefit from the medication ticlopidine hydrochloride (Ticlid®). 3. is experiencing pain that is psychological in origin. 4. needs further medical consultation to determine the cause of pain

4\ The client requires further medical consultation because the ABI (comparison of blood pressure in ankle to the brachial blood pressure) is normal in each leg. A result of less than 0.9 is diagnostic of peripheral arterial disease. Based on the results of the ABI, the client has normal arterial circulation and would not benefit from ticlopidine hydrochloride. There is no information relating the pain to a psychological concern.

The nurse is preparing to teach a client with microcytic hypochromic anemia about the diet to follow after discharge. Which of the following foods should be included in the diet? A Eggs B Lettuce C Citrus fruits D Cheese

A One of the microcytic, hypochromic anemias is iron-deficiency anemia. A rich source of iron is needed in the diet, and eggs are high in iron. Other foods high in iron include organ and muscle (dark) meats; shellfish, shrimp, and tuna; enriched, whole-grain, and fortified cereals and breads; legumes, nuts, dried fruits, and beans; oatmeal; and sweet potatoes. Dark green leafy vegetables and citrus fruits are good sources of vitamin C. Cheese is a good source of calcium.

A vegetarian client was referred to a dietitian for nutritional counseling for anemia. Which client outcome indicates that the client does not understand nutritional counseling? The client: A Adds dried fruit to cereal and baked goods B Cooks tomato-based foods in iron pots C Drinks coffee or tea with meals D Adds vitamin C to all meals

C Coffee and tea increase gastrointestinal motility and inhibit the absorption of nonheme iron. Clients are instructed to add dried fruits to dishes at every meal because dried fruits are a nonheme or nonanimal iron source. Cooking in iron cookware, especially acid-based foods such as tomatoes, adds iron to the diet. Clients are instructed to add a rich supply of vitamin C to every meal because the absorption of iron is increased when food with vitamin C or ascorbic acid is consumed.

Which of the following nursing assessments is a late symptom of polycythemia vera? A Headache B Dizziness C Pruritus D Shortness of breath

C Pruritus is a late symptom that results from abnormal histamine metabolism. Headache and dizziness are early symptoms from engorged veins. Shortness of breath is an early symptom from congested mucous membrane and ineffective gas exchange.

22. An African-American woman had experienced severe palpitations, weakness, and shortness of breath after taking bacitracin (Bactrim). As a part of the discharge planning, the nurse should evaluate the client's knowledge about: ■ 1. Increased folic acid needs. ■ 2. Congenital enzyme defi ciency. ■ 3. Restricted activity in hot weather. ■ 4. Need for blood transfusions.

This client presented with the typical signs of glucose-6-phosphate dehydrogenase (G6PD)-defi ciency anemia. Ten percent of African Americans inherit an X-linked recessive disorder of the G6PD enzyme in the red blood cell (RBC). When cells with decreased levels of G6PD are exposed to certain drugs, such as sulfonamides, acetylsalicylic acid, thiazide diuretics, and vitamin K, the RBC may hemolyze and anemia and jaundice may occur. The reaction is self-limited as soon as the causative agent is withheld. No further treatment is necessary except counseling to prevent acute incidence by avoiding exposure to specifi c drugs. There is no need for increased folic acid, restricted activity in hot weather, or blood transfusions.

82. Which instructions should the nurse include when developing a teaching plan for a client being discharged from the hospital on anticoagulant therapy after having deep vein thrombosis (DVT)? Select all that apply. ■ 1. Checking urine for bright blood and a dark smoky color. ■ 2. Daily walking as a good exercise. ■ 3. Using garlic and ginger, which may decrease bleeding time. ■ 4. Performing foot/leg exercises and walking around the airplane cabin on long fl ights. ■ 5. Prevention as the best treatment for DVT. ■ 6. Avoiding surface bumps because the skin is prone to injury.

1, 2, 4, 5, 6. Clients with resolving DVT being sent home on anticoagulant therapy need instructions about assessing and preventing bleeding episodes and preventing a recurrence of DVT. Blood in the urine (hematuria) is often one of the fi rst symptoms of anticoagulant overdose. Fresh blood in the urine is red; however, blood in the urine may also be a dark smoky color. Daily ambulation is an excellent activity to keep the venous blood circulating and thus to prevent blood clots from forming in the lower extremities. Garlic and ginger increase the bleeding time and should not be used when a client is on anticoagulant therapy. Clients who have had previous DVTs should avoid activities that cause stagnation and pooling of venous blood. Prolonged sitting coupled with change of air pressure without foot or leg exercises or ambulation in the cabin are activities that prevent venous return. Instructing the client about prevention measures is important because clients with DVT are at high risk for pulmonary emboli (PE), which can be fatal. The client can be taught risk factors for DVT and PE. In addition, recommendations for prevention of these events also are standard protocol in practice and should be shared with the client for home care purposes. Older adults should be monitored closely for bleeding because the skin becomes thinner and the capillaries become more fragile with the aging process.

6. A common abnormal laboratory result associated with the development of peripheral vascular disease (PVD) is: ■ 1. High serum calcium level. ■ 2. High serum lipid levels. ■ 3. Low serum potassium level. ■ 4. Low serum lipid levels.

2. High serum lipid levels are associated with an increased incidence of PVD. High serum calcium level, low serum potassium level, and low serum lipid levels have no relation to PVD.

108. When performing external chest compressions on an adult during cardiopulmonary resuscitation (CPR), the rescuer should depress the sternum: ■ 1. 0.5 to 1 inch. ■ 2. 1 to 1.5 inches. ■ 3. 1.5 to 2 inches. ■ 4. 2 to 2.5 inches.

3. An adult's sternum must be depressed 1.5 to 2 inches with each compression to ensure adequate heart compression.

A client admitted with unstable angina is started on intravenous heparin and nitroglycerin. The client's chest pain resolves, and the client is weaned from the nitroglycerin. Noting that the client had a synthetic valve replacement for aortic stenosis 2 years ago, a physician writes an order to restart the oral warfarin (Coumadin®) 5 mg at 1900 hours. Which is the nurse's best action? 1. Administer the warfarin as prescribed. 2. Call the physician to question the warfarin order. 3. Discontinue the heparin drip and then administer the warfarin. 4. Hold the dose of warfarin until the heparin has been discontinued. ANSWER: 1 Both heparin and warfarin are anticoagulants, but their actions are different. Oral warfarin requires 3 to 5 days to reach effective levels. It is usually begun while the client is still on heparin. Calling the physician is unnecessary. The nurse's scope of practice does not permit altering medication

1 Both heparin and warfarin are anticoagulants, but their actions are different. Oral warfarin requires 3 to 5 days to reach effective levels. It is usually begun while the client is still on heparin. Calling the physician is unnecessary. The nurse's scope of practice does not permit altering medication orders. The nurse should neither discontinue the heparin nor hold the warfarin without a written order.

51. Which of the following nursing diagnoses would be appropriate for a client with systolic heart failure? Select all that apply. ■ 1. Ineffective peripheral tissue perfusion related to a decreased stroke volume. ■ 2. Activity intolerance related to impaired gas exchange and perfusion. ■ 3. Dyspnea related to pulmonary congestion and impaired gas exchange. ■ 4. Decreased cardiac output related to impaired cardiac fi lling. ■ 5. Impaired renal perfusion related to a decreased cardiac output.

1, 2, 3, 5. A decrease in cardiac output occurs from a decreased stroke volume with impaired contractility in systolic heart failure. This impairs peripheral and renal perfusion. The impaired perfusion and impaired oxygenation cause the symptoms of activity intolerance. The decreased systolic function causes an increase in residual volume and pressure in the left ventricle. A retrograde buildup of pressure from the left ventricle to left atria increases hydrostatic pressure in the pulmonary vasculature. This causes a leakage of fl uid into the interstitial tissue of the lungs resulting in pulmonary symptoms. With diastolic heart failure, there is impaired ventricular fi lling due to a rigid ventricle and reduced ventricular relaxation`.

3. An overweight client taking warfarin (Coumadin) has a nursing diagnosis of Ineffective tissue perfusion related to decreased arterial blood fl ow. What should the nurse instruct the client to do? Select all that apply. ■ 1. Apply lanolin or petroleum jelly to intact skin. ■ 2. Encourage a reduced-calorie, reduced-fat diet. ■ 3. Inspect the involved areas daily for new ulcerations. ■ 4. Instruct the client to limit activities of daily living (ADLs). ■ 5. Use an electric razor to shave.

1, 2, 3, 5. Maintaining skin integrity is important in preventing chronic ulcers and infections. The client should be taught to routinely inspect the skin on a daily basis. The client should reduce weight to promote circulation; a diet lower in calories and fat is appropriate. Because the client is receiving Coumadin, the client is at risk for bleeding from cuts. To decrease the risk of cuts, the nurse should suggest that the client use an electric razor. The client with decreased arterial blood fl ow should be encouraged to participate in ADLs. In fact, the client should be encouraged to consult an exercise physiologist for an exercise program that enhances the aerobic capacity of the body.

4. A client is to have a transfusion of packed red blood cells from a designated donor. The client asks if any diseases can be transmitted by this donor. The nurse should inform the client that which of the following diseases can be transmitted by a designated donor? Select all that apply. ■ 1. Epstein-Barr virus. ■ 2. Human immunodefi ciency virus (HIV). ■ 3. Cytomegalovirus (CMV). ■ 4. Hepatitis A. ■ 5. Malaria.

1, 2, 3. Using designated donors does not decrease the risk of contracting infectious diseases, such as the Epstein-Barr virus, HIV, or CMV. Hepatitis A is transmitted by the oral-fecal route, not the blood route; however, hepatitis B and C can be contracted from a designated donor. Malaria is transmitted by mosquitoes.

2. A client with iron defi ciency anemia is refusing to take the prescribed oral iron medication because the medication is causing nausea. The nurse should do which of the following? Select all that apply. ■ 1. Suggest that the client use ginger when taking the medication. ■ 2. Ask the client what she thinks is causing the nausea. ■ 3. Tell the client to use stool softeners to minimize constipation. ■ 4. Offer to administer the medication by an intramuscular injection. ■ 5. Suggest that the client take the iron with orange juice.

1, 2, 5. Nausea and vomiting are common adverse effects of oral iron preparations. The nurse should fi rst ask the client why she does not want to take the oral medication, and then suggest ways to decrease the nausea and vomiting. Ginger may help minimize the nausea and the client can try this remedy and evaluate its effectiveness. Iron should be taken on an empty stomach but can be taken with orange juice. The client can evaluate if this helps the nausea. Stool softeners should not be used in clients with iron defi ciency anemia. Instead, constipation can be prevented by following a high-fi ber diet. Administering iron intramuscularly is done only if other approaches are not effective.

53. A client has been diagnosed with vasospastic disorder (Raynaud's phenomenon) on the tip of the nose and fi ngertips. The physician has prescribed reserpine (Serpasil) to determine if the client will obtain relief. The client's history reveals that he lives in Vermont and works outside in the logging industry. He smokes two packs of cigarettes per day. Which of the following components are an important part of the discharge plan for this client? Select all that apply. ■ 1. Stopping smoking. ■ 2. Wearing a face covering and gloves in the winter. ■ 3. Placing fi ngertips in cool water to rewarm them. ■ 4. Finding employment that can be done in a warm environment. ■ 5. Reporting signs of orthostatic hypotension

1, 2. Vasospastic disorder (Raynaud's disease) is a form of intermittent arteriolar vasoconstriction that results in coldness, pain, and pallor of the fi ngertips, toes, or tip of the nose, and a rebound circulation with redness and pain. The nurse should instruct the client to stop smoking because nicotine is a vasoconstrictor. An adverse effect of reserpine is orthostatic hypotension. The client should report dizziness and low blood pressure as it may be necessary to consider stopping the drug. The client should prevent vasoconstriction by covering affected parts when in cold environments. The nurse can teach the client to rewarm exposed extremities by using warm water or placing them next to the body, such as under the axilla. It is not realistic to ask this client to change jobs at this time

104. The nurse is preparing the client for cardioversion. The nurse should do which of the following? Select all that apply. ■ 1. Use a conducting agent between the skin and the paddles. ■ 2. Place the paddles over the client's clothing. ■ 3. Call "clear" before discharging the electrical current. ■ 4. Record the delivered energy and the resulting rhythm. ■ 5. Exert 5 to 10 lb of pressure on each paddle to ensure good skin contact.

1, 3, 4. A conducting agent is placed between the skin and the paddles to conduct the electrical current when discharged. The nurse must make sure to call "clear" before discharging the electrical current to prevent injury to others who may be helping with the client. Each paddle is placed directly on the conductive pads that are on the client's skin. Applying approximately 20 to 25 lb of pressure on each paddle is recommended to ensure good skin contact. The nurse must record the amount of electrical current delivered and the resulting rhythm.

84. A client is taking clonidine (Catapres) for treatment of hypertension. The nurse should teach the client about which of the following common adverse effects of this drug? Select all that apply. ■ 1. Dry mouth. ■ 2. Hyperkalemia. ■ 3. Impotence. ■ 4. Pancreatitis. ■ 5. Sleep disturbance.

1, 3, 5. Clonidine (Catapres) is a centralacting adrenergic antagonist. It reduces sympathetic outfl ow from the central nervous system. Dry mouth, impotence, and sleep disturbances are possible adverse effects. Hyperkalemia and pancreatitis are not anticipated with use of this drug.

A nurse is assessing a client with heart failure. The nurse should assess the client based on which compensatory mechanisms that are activated in the presence of heart failure? Select all that apply. ■ 1. Ventricular hypertrophy. ■ 2. Parasympathetic nervous stimulation. ■ 3. Renin-angiotensin-aldosterone system. ■ 4. Jugular venous distention. ■ 5. Sympathetic nervous stimulation.

1, 3, 5. When the heart begins to fail, the body activates three major compensatory systems: ventricular hypertrophy, the renin-angiotensinaldosterone system, and sympathetic nervous stimulation. Parasympathetic stimulation and jugular venous distention are not compensatory mechanisms associated with heart failure.

4. A client with acute chest pain is receiving I.V. morphine sulfate. Which of the following results are intended effects of morphine in this client? Select all that apply. ■ 1. Reduces myocardial oxygen consumption. ■ 2. Promotes reduction in respiratory rate. ■ 3. Prevents ventricular remodeling. ■ 4. Reduces blood pressure and heart rate. ■ 5. Reduces anxiety and fear.

1, 4, 5. Morphine sulfate acts as an analgesic and sedative. It also reduces myocardial oxygen consumption, blood pressure, and heart rate. Morphine also reduces anxiety and fear due to its sedative effects and by slowing the heart rate. It can depress respirations; however, such an effect may lead to hypoxia, which should be avoided in the treatment of chest pain. Angiotensin-converting enzyme- inhibitor drugs, not morphine, may help to prevent ventricular remodeling.

98. When teaching a client about self-care following placement of a new permanent pacemaker to his left upper chest, the nurse should include which information? Select all that apply. ■ 1. Take and record daily pulse rate. ■ 2. Avoid air travel because of airport security alarms. ■ 3. Immobilize the affected arm for 4 to 6 weeks. ■ 4. Avoid using a microwave oven. ■ 5. Avoid lifting anything heavier than 3 lb.

1, 5. The nurse must teach the client how to take and record his pulse daily. The client should be instructed to avoid lifting the operative-side arm above shoulder level for 1 week postinsertion. It takes up to 2 months for the incision site to heal and full range of motion to return. The client should avoid heavy lifting until approved by the physician. The pacemaker metal casing does not set off airport security alarms, so there are no travel restrictions. Prolonged immobilization is not required. Microwave ovens are safe to use and do not alter pacemaker function.

5. Blood pressure in the systemic circulation is highest in the: ■ 1. Arterioles. ■ 2. Capillaries. ■ 3. Aorta. ■ 4. Venules

3. Blood pressure is the highest in the aorta as the blood is being ejected out of the left ventricle into the aorta. The pressure declines as the blood fl ows through the arteries, capillaries, arterioles, veins, capillaries, and venules. The force of the contraction of the heart and resistance of vessels infl uence fl ow; however, it is the pressure differences that control blood fl ow.

9. A client is admitted with a myocardial infarction and new onset atrial fi brillation. While auscultating the heart, the nurse notes an irregular heart rate and hears an extra heart sound at the apex after the S2 that remains constant throughout the respiratory cycle. The nurse should document these fi ndings as: ■ 1. Heart rate irregular with S3. ■ 2. Heart rate irregular with S4. ■ 3. Heart rate irregular with aortic regurgitation. ■ 4. Heart rate irregular with mitral stenosis.

1. An S3 heart sound occurs early in diastole as the mitral and tricuspid valves open and blood rushes into the ventricles. To distinguish an S3 from a physiologic S2 split, a split S2 occurs during inspiration and S3 remains constant during the respiratory cycle. Its pitch is softer and best heard with the bell at the apex and it is one of the fi rst clinical fi ndings in left ventricular failure. An S4 is heard in late diastole when atrial contraction pumps volume into a stiff, noncompliant ventricle. An S4 is not heard in a client with atrial fi brillation because there is no atrial contraction. Murmurs are sounds created by turbulent blood fl ow through an incompetent or stenotic valve.

26. A client is scheduled for an arteriogram. The nurse should explain to the client that the arteriogram will confi rm the diagnosis of occlusive arterial disease by: ■ 1. Showing the location of the obstruction and the collateral circulation. ■ 2. Scanning the affected extremity and identifying the areas of volume changes. ■ 3. Using ultrasound to estimate the velocity changes in the blood vessels. ■ 4. Determining how long the client can walk

1. An arteriogram involves injecting a radiopaque contrast agent directly into the vascular system to visualize the vessels. It usually involves computed tomographic scanning. The velocity of the blood fl ow can be estimated by duplex ultrasound. The client's ankle-brachial index is determined, and then the client is requested to walk. The normal response is little or no drop in ankle systolic pressure after exercise

20. One goal of care for a client with PVD is to decrease anxiety, so as to decrease or prevent vasoconstriction of the: ■ 1. Arteries. ■ 2. Capillaries. ■ 3. Lymphatics. ■ 4. Veins.

1. Anxiety stimulates the sympathetic nervous system, which results in the secretion of epinephrine, angiotensin, and serum proteins that cause vasoconstriction in the arteries of the peripheral circulatory system. As a result, peripheral vascular resistance is increased. This vasoconstriction may increase pain in the areas where PVD is the greatest. The lymphatic system does not affect the blood supply of tissues.

80. Good dental care is an important measure in reducing the risk of endocarditis. A teaching plan to promote good dental care in a client with mitral stenosis should include demonstration of the proper use of: ■ 1. A manual toothbrush. ■ 2. An electric toothbrush. ■ 3. An irrigation device. ■ 4. Dental fl oss.

1. Daily dental care and frequent checkups by a dentist who is informed about the client's condition are required to maintain good oral health. Use of an electric toothbrush, an irrigation device, or dental fl oss may cause gums to bleed and allow bacteria to enter mucous membranes and the bloodstream, increasing the risk of endocarditis

89. Before surgery for a known aortic aneurysm, the client's pulse pressure begins to widen, suggesting increased aortic valvular insuffi ciency. If the branches of the aortic arch are involved, the nurse should assess the client for: ■ 1. Loss of consciousness. ■ 2. Anxiety. ■ 3. Headache. ■ 4. Disorientation.

1. If the aortic arch is involved, there will be a decrease in the blood fl ow to the cerebrum. Therefore, loss of consciousness will be observed. A sudden loss of consciousness is a primary symptom of rupture and no blood fl ow to the brain. Anxiety is not a sign of aortic valvular insuffi ciency. The end result of decreased cerebral blood fl ow is loss of consciousness, not headache or disorientation.

78. For a client who excretes excessive amounts of calcium during the postoperative period after open heart surgery, which of the following measures should the nurse institute to help prevent complications associated with excessive calcium excretion? ■ 1. Ensure a liberal fl uid intake. ■ 2. Provide an alkaline-ash diet. ■ 3. Prevent constipation. ■ 4. Enrich the client's diet with dairy products

1. In an immobilized client, calcium leaves the bone and concentrates in the extracellular fl uid. When a large amount of calcium passes through the kidneys, calcium can precipitate and form calculi. Nursing interventions that help prevent calculi include ensuring a liberal fl uid intake (unless contraindicated). A diet rich in acid should be provided to keep the urine acidic, which increases the solubility of calcium. Preventing constipation is not associated with excessive calcium excretion. Limiting foods rich in calcium, such as dairy products, will help in preventing renal calculi.

53. The major goal of therapy for a client with heart failure and pulmonary edema should be to: ■ 1. Increase cardiac output. ■ 2. Improve respiratory status. ■ 3. Decrease peripheral edema. ■ 4. Enhance comfort

1. Increasing cardiac output is the main goal of therapy for the client with heart failure or pulmonary edema. Pulmonary edema is an acute medical emergency requiring immediate intervention. Respiratory status and comfort will be improved when cardiac output increases to an acceptable level. Peripheral edema is not typically associated with pulmonary edema.

50. A 69-year-old female has a history of heart failure. She is admitted to the emergency department with heart failure complicated by pulmonary edema. On admission of this client, which of the following should the nurse assess fi rst? ■ 1. Blood pressure. ■ 2. Skin breakdown. ■ 3. Serum potassium level. ■ 4. Urine output.

1. It is a priority to assess blood pressure fi rst because people with pulmonary edema typically experience severe hypertension that requires early intervention. The client probably does not have skin breakdown on admission; however, when the client is stable, the nurse should inspect the skin. Potassium levels are not the fi rst priority. The nurse should monitor urine output after the client is stable.

6. The nurse is preparing to teach a client with microcytic hypochromic anemia about the diet to follow after discharge. Which of the following foods should be included in the diet? ■ 1. Eggs. ■ 2. Lettuce. ■ 3. Citrus fruits. ■ 4. Cheese.

1. One of the microcytic, hypochromic anemias is iron defi ciency anemia. A rich source of iron is needed in the diet, and eggs are high in iron. Other foods high in iron include organ and muscle (dark) meats; shellfi sh, shrimp, and tuna; enriched, whole-grain, and fortifi ed cereals and breads; legumes, nuts, dried fruits, and beans; oatmeal; and sweet potatoes. Dark green, leafy vegetables and citrus fruits are good sources of vitamin C. Cheese is a good source of calcium.

100. The nurse is caring for a client who has just had an ankle-brachial index (ABI) test. The left arm blood pressure was 160/80 mm Hg and a palpable systolic blood pressure of the left lower extremity was 130/60 mm Hg. These fi ndings suggest that the client has: ■ 1. Mild peripheral artery disease. ■ 2. Moderate peripheral artery disease. ■ 3. No apparent occlusion in the left lower extremity. ■ 4. Severe peripheral artery disease.

1. The ABI test is a noninvasive test that compares the systolic blood pressure in the arm with that of the ankle. It may be done before or after exercise. The client's highest brachial systolic pressure is divided by the left ankle systolic blood pressure to get 0.81. This score is between 0.71 and 0.90, which suggests mild peripheral artery disease. Moderate peripheral artery disease would yield a score of 0.41 to 0.70. Severe peripheral artery disease would result in a score of 0.00 to 0.40.

3. A client had a mastectomy followed by chemotherapy 6 months ago. She reports that she is now "unable to concentrate at her card game" and "it seems harder and harder to fi nish her errands because of exhaustion." Based on this information, the nurse should suggest that the client do which of the following? ■ 1. Take frequent naps. ■ 2. Limit activities. ■ 3. Increase fl uid intake. ■ 4. Avoid contact with others.

1. This client is likely experiencing fatigue and should increase her periods of rest. The fatigue may be caused by anemia from depletion of red blood cells due to the chemotherapy. Asking the client to limit her activities may cause the client to become withdrawn. The information given does not support limiting activity. Increasing fl uid intake will not reduce the fatigue. The information does not indicate that the client is immunosuppressed and should avoid contact with others.

99. A client has been admitted to the coronary care unit. The nurse observes third-degree heart block at a rate of 35 bpm on the client's cardiac monitor. The client has a blood pressure of 90/60. The nurse should take which of the following actions fi rst? ■ 1. Prepare for transcutaneous pacing. ■ 2. Prepare to defi brillate the client at 200 joules. ■ 3. Administer an I.V. lidocaine infusion. ■ 4. Schedule the operating room for insertion of a permanent pacemaker.

1. Transcutaneous pacemaker therapy provides an adequate heart rate to a client in an emergency situation. Defi brillation and a lidocaine infusion are not indicated for the treatment of thirddegree heart block. Transcutaneous pacing is used temporarily until a transvenous or permanent pacemaker can be inserted.

65. A nurse is assigned to a client with venous thrombus. The nurse identifi es a nursing diagnosis of Impaired physical mobility related to pain. Which should the nurse do fi rst? ■ 1. Elevate the legs. ■ 2. Elevate the legs by using a pillow under the knees. ■ 3. Encourage adequate fl uid intake. ■ 4. Massage the lower legs.

1. Venous stasis can increase pain. Therefore, proper positioning in bed or when sitting up in a chair can help promote venous drainage, reduce swelling, and reduce the amount of pain the client might experience. Placing a pillow under the knees causes fl exion of the joint, resulting in a dependent position of the lower leg and causing a decrease in blood fl ow. Fluids are encouraged to maintain normal fl uid and electrolyte balance but do little to relieve pain. Therapeutic massage to the legs is discouraged because of the danger of breaking up the clot.

82. Three days after mitral valve surgery, a 45-year-old female comments that she hears a "clicking" noise coming from her chest and her "rather large" chest incision. The nurse's response should refl ect the understanding that the client may be experiencing which of the following? ■ 1. Anxiety related to altered body image. ■ 2. Anxiety related to altered health status. ■ 3. Altered tissue perfusion. ■ 4. Lack of knowledge regarding the postoperative course.

1. Verbalized concerns from this client may stem from her anxiety over the changes her body has gone through after open heart surgery. Although the client may experience anxiety related to her altered health status or may have a lack of knowledge regarding her postoperative course, she is pointing out the changes in her body image. The client is not concerned about altered tissue perfusion.

A nurse evaluates that a client understands discharge teaching, following aortic valve replacement surgery with a synthetic valve, when the client states that he/she plans to: SELECT ALL THAT APPLY. 1. use a soft toothbrush for dental hygiene. 2. floss teeth daily to prevent plaque formation. 3. wear loose-fitting clothing to avoid friction on the sternal incision. 4. use an electric razor for shaving. 5. report black, tarry stools. 6. consume foods high in vitamin K, such as broccoli

1345 A synthetic heart valve requires long-term anticoagulation because of the risk of thromboembolism. Because low-dose aspirin, which prevents platelet aggregation, and oral anticoagulation together are more effective than just oral anticoagulation to reduce the risk of thromboembolism after valve replacement, both are prescribed, which increases the risk for bleeding. Bleeding precautions while on anticoagulation include using a soft toothbrush, avoiding injury (such as can occur with flossing), and using an electric razor. The client will have a sternal incision. Care must be taken to avoid tissue trauma. Black, tarry stools are a sign of bleeding. Flossing should be avoided because it causes tissue trauma, increases the risk of bleeding, and increases the risk of infective endocarditis. The diet should contain normal amounts of vitamin K; excessive amounts antagonize the effects of the anticoagulant.

A nurse is planning care for a client admitted with a new diagnosis of persistent atrial fibrillation with rapid ventricular response. Although the client has had no previous cardiac problems, the client has been in atrial fibrillation for more than 2 days. The nurse should anticipate that the health-care provider is likely to initially order: SELECT ALL THAT APPLY. 1. oxygen. 2. immediate cardioversion. 3. administration of amiodarone (Cordarone®). 4. initiation of a IV heparin infusion. 5. immediate catheter-directed ablation of the AV node. 6. administration of a calcium channel antagonist such as diltiazem (Cardizem®).

1346 The ineffective atrial contractions or loss of atrial kick with atrial fibrillation can decrease cardiac output. Administering oxygen enhances tissue oxygenation. Amiodarone is used for pharmacological cardioversion of the atrial fibrillation rhythm. The client is at risk for thrombi in the atria from stasis. Anticoagulant therapy is used to prevent thromboembolism. Diltiazem, a calcium channel antagonist, is prescribed to slow the ventricular response to atrial fibrillation. An alternative to a calcium channel antagonist would be the use of a beta blocker, such as esmolol, metoprolol, or propranolol. Cardioversion would only be considered if medications were ineffective in converting the client's rhythm and only after the presence of an atrial clot has been ruled out. Ablation of the AV node would only be considered if medications were ineffective in controlling the client's heart rate.

A nurse receives a serum laboratory report for six different clients with admitting diagnoses of chest pain. After reviewing all of the lab reports, in which order should the nurse address each lab value? Prioritize the order in which the nurse should address each of the clients' results. ______ Troponin T 42 ng/mL (0.0-0.4 ng/mL) ______ WBC 11,000 K/μL ______ Hgb 7.2 g/dL ______ SCr 2.2 mg/dL ______ K 2.2 mEq/L ______ Total cholesterol 430 mg/dL

163425 The nurse should address the elevated troponin level first. Cardiospecific troponins (troponin T, cTnT, and troponin I, cTnI) are released into circulation after myocardial injury and are highly specific indicators of myocardial infarction. Since "time is muscle," the client needs to be treated immediately to prevent extension of the infarct and possible death. The nurse should address the decreased serum potassium level (K) second. The normal serum K level is 3.5 to 5.8 mEq/L. A low serum K level can cause life-threatening dysrhythmias. The normal hemoglobin (Hgb) is 13.1 to 17.1 g/dL. A low Hgb can contribute to inadequate tissue perfusion and contribute to myocardial ischemia. The normal serum creatinine (SCr) is 0.4 to 1.4 mg/dL. Impaired circulation may be causing this alteration and further client assessment is needed. Medication doses may need to be adjusted with impaired renal perfusion. The normal total serum cholesterol should be less than 200 mg/dL. This is a risk factor for development of coronary artery disease. The client needs teaching. The normal white blood cell (WBC) count is 3.9 to 11.9 K/μL. Because the finding is normal, it can be addressed last

40. Which of the following is not a risk factor for the development of atherosclerosis? ■ 1. Family history of early heart attack. ■ 2. Late onset of puberty. ■ 3. Total blood cholesterol level greater than 220 mg/dL. ■ 4. Elevated fasting blood glucose concentration.

2. Late onset of puberty is not generally considered to be a risk factor for the development of atherosclerosis. Risk factors for atherosclerosis include family history of atherosclerosis, cigarette smoking, hypertension, high blood cholesterol level, male gender, diabetes mellitus, obesity, and physical inactivity.

70. A client has returned to the medical-surgical unit after a cardiac catheterization. Which is the most important initial postprocedure nursing assessment for this client? ■ 1. Monitor the laboratory values. ■ 2. Observe neurologic function every 15 minutes. ■ 3. Observe the puncture site for swelling and bleeding. ■ 4. Monitor skin warmth and turgor.

3. Assessment of circulatory status, including observation of the puncture site, is of primary importance after a cardiac catheterization. Laboratory values and skin warmth and turgor are important to monitor but are not the most important initial nursing assessment. Neurologic assessment every 15 minutes is not required.

A client experiences cardiac arrest at home and is successfully resuscitated. Following placement of an implantable cardioverter-defibrillator (ICD), a nurse is evaluating the effectiveness of teaching for the client. Which statement, if made by the client, indicates that further teaching is needed? 1. "The ICD will monitor my heart activity and provide a shock to my heart if my heart goes into ventricular fibrillation again." 2. "When I feel the first shock I should tell my family to start cardiopulmonary resuscitation (CPR) and call 911." 3. "I am fearful of my first shock since my friend stated his shock felt like a blow to the chest." 4. "I will need to ask my physician when I can resume driving because some states disallow driving until there is a 6-month discharge-free period."

2 If the first shock is unsuccessful, the device will recycle and continue to deliver shocks. If the device fires more than once, the emergency medical services (EMS) system should be activated. The ICD continues to deliver shocks if indicated; CPR should only be initiated after the shocks have been delivered and only if the client is unresponsive and pulseless. The ICD monitors the client's heart rate and rhythm, identifies ventricular tachycardia or ventricular fibrillation, and delivers a 25-joule or less shock if a lethal rhythm is detected. Various sensations have been described when the device delivers a shock, including a blow to or kick in the chest. State laws vary regarding drivers with ICDs. The decision regarding driving is also based on whether dysrhythmias are present, the frequency of firing, and the client's overall health.

62. A client receiving a loop diuretic should be encouraged to eat which of the following foods? Select all that apply. ■ 1. Angel food cake. ■ 2. Banana. ■ 3. Dried fruit. ■ 4. Orange juice. ■ 5. Peppers.

2, 3, 4. Hypokalemia is a side effect of loop diuretics. Bananas, dried fruit, and oranges are examples of food high in potassium. Angel food cake, yellow cake, and peppers are listed by the National Kidney Foundation as low in potassium.

86. In teaching the hypertensive client to avoid orthostatic hypotension, the nurse should emphasize which of the following instructions? Select all that apply. ■ 1. Plan regular times for taking medications. ■ 2. Arise slowly from bed. ■ 3. Avoid standing still for long periods. ■ 4. Avoid excessive alcohol intake. ■ 5. Avoid hot baths.

2, 3. Changing positions slowly and avoiding long periods of standing may limit the occurrence of orthostatic hypotension. Scheduling regular medication times is important for blood pressure management but this aspect is not related to the development of orthostatic hypotension. Excessive alcohol intake and hot baths are associated with vasodilation.

7. The nurse has completed an assessment on a client with a decreased cardiac output. Which fi ndings should receive the highest priority? ■ 1. BP 110/62, atrial fi brillation with HR 82, bibasilar crackles. ■ 2. Confusion, urine output 15 mL over the last 2 hours, orthopnea. ■ 3. SpO2 92 on 2 liters nasal cannula, respirations 20, 1+ edema of lower extremities. ■ 4. Weight gain of 1 kg in 3 days, BP 130/80, mild dyspnea with exercise.

2. A low urine output and confusion are signs of decreased tissue perfusion. Orthopnea is a sign of left-sided heart failure. Crackles, edema and weight gain should be monitored closely, but the levels are not as high a priority. With atrial fi brillation there is a loss of atrial kick, but the blood pressure and heart rate are stable.

111. The rapid response team has been called to manage an unwitnessed cardiac arrest. The estimated maximum time a person can be without cardiopulmonary function and still not experience permanent brain damage is: ■ 1. 1 to 2 minutes. ■ 2. 4 to 6 minutes. ■ 3. 8 to 10 minutes. ■ 4. 12 to 15 minutes.

2. After a person is without cardiopulmonary function for 4 to 6 minutes, permanent brain damage is almost certain. To prevent permanent brain damage, it is important to begin CPR promptly after a cardiopulmonary arrest.

10. The nurse devises a teaching plan for the client with aplastic anemia. Which of the following is the most important concept to teach for health promotion and maintenance? ■ 1. Eat animal protein and dark green, leafy vegetables every day. ■ 2. Avoid exposure to others with acute infections. ■ 3. Practice yoga and meditation to decrease stress and anxiety. ■ 4. Get 8 hours of sleep at night and take naps during the day.

2. Clients with aplastic anemia are severely immunocompromised and at risk for infection and possible death related to bone marrow suppression and pancytopenia. Strict aseptic technique and reverse isolation are important measures to prevent infection. Although diet, reduced stress, and rest are valued in supporting health, the potentially fatal consequence of an acute infection places it as a priority for teaching the client about health maintenance. Animal meat and dark green leafy vegetables, good sources of vitamin B12 and folic acid, should be included in the daily diet. Yoga and meditation are good complementary therapies to reduce stress. Eight hours of rest and naps are good for spacing and pacing activity and rest.

72. A client experiences initial indications of excitation after having an I.V. infusion of lidocaine hydrochloride started. The nurse should further assess the client when the client reports having: ■ 1. Palpitations. ■ 2. Tinnitus. ■ 3. Urinary frequency. ■ 4. Lethargy.

2. Common adverse effects of lidocaine hydrochloride include dizziness, tinnitus, blurred vision, tremors, numbness and tingling of extremities, excessive perspiration, hypotension, seizures, and fi nally coma. Cardiac effects include slowed conduction and cardiac arrest. Palpitations, urinary frequency, and lethargy are not considered typical adverse reactions to lidocaine.

57. During an initial assessment of a client diagnosed with vasospastic disorder (Raynaud's phenomenon), the nurse notes a sudden color change to white in the fi ngers. The nurse should fi rst assess: ■ 1. Appearance of cyanosis. ■ 2. Radial pulse. ■ 3. SpO2 of the affected fi ngers. ■ 4. Blood pressure.

2. Decreased perfusion from vasospasm induces color changes in the extremity. The degree of decreased perfusion should be assessed by taking the radial pulse. Color changes progressively to blue with cyanosis and then red when reperfusion occurs. The SpO2 requires adequate perfusion for accuracy. A blood pressure will cause further constriction and reduction of perfusion in the extremity

13. A client admitted for a myocardial infarction (MI) develops cardiogenic shock. An arterial line is inserted. Which of the following orders should the nurse question? ■ 1. Call for urine output < 30 mL/hour for 2 consecutive hours. ■ 2. Metoprolol (Lopressor) 5 mg I.V. push. ■ 3. Prepare for a pulmonary artery catheter insertion. ■ 4. Titrate Dobutamine (Dobutrex) to keep systolic BP > 100.

2. Metoprolol is indicated in the treatment of hemodynamically stable clients with an acute MI to reduce cardiovascular mortality. Cardiogenic shock causes severe hemodynamic instability and a beta blocker will further depress myocardial contractility. The metoprolol should be discontinued. The decrease in cardiac output will impair perfusion to the kidneys. Cardiac output, hemodynamic measurements, and appropriate interventions can be determined with a PA catheter. Dobutamine will improve contractility and increase the cardiac output that is depressed in cardiogenic shock.

67. A client is discharged after being hospitalized for thrombophlebitis. She will be driving home with her daughter, who lives 2 hours away. During the 2-hour ride, the nurse should advise the client to: ■ 1. Perform arm circles while riding in the car. ■ 2. Perform ankle pumps and foot range-ofmotion exercises. ■ 3. Elevate her legs while riding in the car. ■ 4. Take an ambulance home.

2. Performing active ankle and foot range-ofmotion exercises periodically during the ride home will promote muscular contraction and provide support to the venous system. It is the muscular action that facilitates return of the blood from the lower extremities, especially when in the dependent position. Arm circle exercises will not promote circulation in the leg. It is not necessary for the client to elevate her legs as long as she does not occlude blood fl ow to her legs and does her leg exercises. It is not necessary to take an ambulance because the client is able to sit in the car safely.

105. A client who has been given cardiopulmonary resuscitation (CPR) is transported by ambulance to the hospital's emergency department, where the admitting nurse quickly assesses the client's condition. The most effective way to determine the effectiveness of CPR is noting whether the: ■ 1. Pulse rate is normal. ■ 2. Pupils are reacting to light. ■ 3. Mucous membranes are pink. ■ 4. Systolic blood pressure is at least 80 mm Hg.

2. Pupillary reaction is the best indication of whether oxygenated blood has been reaching the client's brain. Pupils that remain widely dilated and do not react to light probably indicate that serious brain damage has occurred. The pulse rate may be normal, mucous membranes may still be pink, and systolic blood pressure may be 80 mm Hg or higher, and serious brain damage may still have occurred.

pulmonary artery catheter is inserted in a client with severe mitral stenosis and regurgitation. The nurse administers furosemide (Lasix) to treat pulmonary congestion and begins a Nitroprusside (Nipride) drip for afterload reduction per physician orders. The nurse notices a sudden drop in the pulmonary artery diastolic pressure and pulmonary artery wedge pressure. Which of the following has the highest priority? ■ 1. Assess the 12-lead EKG. ■ 2. Assess the blood pressure. ■ 3. Assess the lung sounds. ■ 4. Assess the urine output.

2. The nurse should immediately assess the blood pressure since Nipride and Lasix can cause severe hypotension from a decrease in preload and afterload. If the client is hypotensive, the Nipride dose should be reduced or discontinued. Urine output should then be monitored to make sure there is adequate renal perfusion. A 12-lead EKG is performed if the client experiences chest pain. A reduction in pulmonary artery pressures should improve the pulmonary congestion and lung sounds

114. The monitor technician informs the nurse that the client has started having premature ventricular contractions every other beat. Which is the priority nursing action? ■ 1. Activate the rapid response team. ■ 2. Assess the client's orientation and vital signs. ■ 3. Call the physician. ■ 4. Administer a bolus of lidocaine.

2. The priority action is to assess the client and determine whether the rhythm is life-threatening. More information, including vital signs, should be obtained and the physician should be quickly notifi ed. A bolus of lidocaine may be ordered to treat this arrhythmia. This is not a code-type situation unless the client has been determined to be in a life-threatening situation.

A nurse is teaching a client newly diagnosed with chronic stable angina. Which instructions should the nurse incorporate in the teaching session on measures to prevent future angina? SELECT ALL THAT APPLY. 1. Increase isometric arm exercises to build endurance. 2. Wear a face mask when outdoors in cold weather. 3. Take nitroglycerin before a stressful situation even though pain is not present. 4. Perform most exertional activities in the morning. 5. Avoid straining at stool. 6. Eliminate tobacco use.

2356 Blood vessels constrict in response to cold and increase the workload of the heart. Sexual activity and straining at stool increases sympathetic stimulation and cardiac workload. Nitroglycerin produces vasodilation and improves blood flow to the coronary arteries; it can be used prophylactically to prevent angina. Nicotine stimulates catecholamine release, producing vasoconstriction and an increased heart rate. Isometric exercise of the arms can cause exertional angina. Exertional activity increases the heart rate, thus reducing the time the heart is in diastole, when blood flow to the coronary arteries is the greatest. A period of rest should occur between activities and activities should be spaced

106. A client is given amiodarone (Cordarone) in the emergency department for a dysrhythmia. Which of the following indicates the drug is having the desired effect? ■ 1. The ventricular rate is increasing. ■ 2. The absent pulse is now palpable. ■ 3. The number of premature ventricular contractions is decreasing. ■ 4. The fi ne ventricular fi brillation changes to coarse ventricular fi brillation.

3. Amiodarone is used for the treatment of premature ventricular contractions, ventricular tachycardia with a pulse, atrial fi brillation, and atrial fl utter. Amiodarone is not used as initial therapy for a pulseless dysrhythmia.

Following a normal chest x-ray for a client who had cardiac surgery, a nurse receives an order to remove the chest tubes. Which intervention should the nurse plan to implement first? 1. Auscultate the client's lung sounds 2. Administer 4 mg morphine sulfate intravenously 3. Turn off the suction to the chest drainage system 4. Prepare the dressing supplies at the client's bedside

2Because the peak action of morphine sulfate is 10 to 15 minutes, this should be administered first. Auscultating the client's lungs before and after the procedure, turning off the suction, and assembling the dressing supplies are all necessary, but administering the analgesic should be first.

A nurse is assessing a client diagnosed with an anterior-lateral myocardial infarction (MI). The nurse adds a nursing diagnosis to the client's plan of care of decreased cardiac output when which finding is noted on assessment? 1. One-sided weakness 2. Presence of an S4 heart sound 3. Crackles auscultated in bilateral lung bases 4. Vesicular breath sounds over lung lobes

3 An anterior-lateral MI can produce left ventricular dysfunction and low cardiac output. With low cardiac output, blood accumulates in the heart and backs up into the pulmonary system. The increased pulmonary pressure causes fluid to move into interstitial spaces and then into the alveoli. One-sided weakness suggests complications of a cerebrovascular accident, which can be caused from a clot or plaque embolus secondary to the MI. An S4 heart sound is produced when blood flows forcefully from the atrium to a resistant ventricle during late ventricular diastole. Vesicular breath sounds are normal over lesser bronchi, bronchioles, and lobes of the lung

client diagnosed with class II heart failure according to the New York Heart Association Functional Classification has been taught about the initial treatment plan for this disease. A nurse determines that the client needs additional teaching if the client states that the treatment plan includes: 1. diuretics. 2. a low-sodium diet. 3. home oxygen therapy. 4. angiotensin-converting enzyme (ACE) inhibitors.

3 In class II heart failure, normal physical activity results in fatigue, dyspnea, palpitations, or anginal pain. The symptoms are absent at rest. Home oxygen therapy is unnecessary unless there are other comorbid conditions. Diuretics mobilize edematous fluid, act on the kidneys to promote excretion of sodium and water, and reduce preload and pulmonary venous pressure. Dietary restriction of sodium aids in reducing edema. ACE inhibitors block the conversion of angiotensin I to the vasoconstrictor angiotensin II, prevent the degradation of bradykinin and other vasodilatory prostaglandins, and increase plasma renin levels and reduce aldosterone levels. The net result is systemic vasodilation, reduced systemic vascular resistance, and improved cardiac output.

91. The client realizes the importance of quitting smoking, and the nurse develops a plan to help the client achieve this goal. Which of the following nursing interventions should be the initial step in this plan? ■ 1. Review the negative effects of smoking on the body. ■ 2. Discuss the effects of passive smoking on environmental pollution. ■ 3. Establish the client's daily smoking pattern. ■ 4. Explain how smoking worsens high blood pressure.

3. A plan to reduce or stop smoking begins with establishing the client's personal daily smoking pattern and activities associated with smoking. It is important that the client understands the associated health and environmental risks, but this knowledge has not been shown to help clients change their smoking behavior.

92. Essential hypertension would be diagnosed in a 40-year-old male whose blood pressure readings were consistently at or above which of the following? ■ 1. 120/90 mm Hg. ■ 2. 130/85 mm Hg. ■ 3. 140/90 mm Hg. ■ 4. 160/80 mm Hg.

3. American Heart Association standards defi ne hypertension as a consistent systolic blood pressure level greater than 140 mm Hg and a consistent diastolic blood pressure level greater than 90 mm Hg.

24. A 58-year-old female with a family history of CAD is being seen for her annual physical exam. Fasting lab test results include: Total cholesterol 198; LDL cholesterol 120; HDL cholesterol 58; Triglycerides 148; Blood sugar 102; and C-reactive protein (CRP) 4.2. The health care provider informs the client that she will be started on a statin medication and aspirin. The client asks the nurse why she needs to take these medications. Which is the best response by the nurse? ■ 1. "The labs indicate severe hyperlipidemia and the medications will lower your LDL, along with a lowfat diet." ■ 2. "The triglycerides are elevated and will not return to normal without these medications." ■ 3. "The CRP is elevated indicating infl ammation seen in cardiovascular disease, which can be lowered by the medications ordered." ■ 4. "The medications are not indicated since your lab values are all normal."

3. CRP is a marker of infl ammation and is elevated in the presence of cardiovascular disease. The high sensitivity CRP (hs-CRP) is the blood test for greater accuracy in measuring the CRP to evaluate cardiovascular risk. The family history, postmenopausal age, LDL above optimum levels and elevated CRP place the client at risk of CAD. Statin medications can decrease LDL, whereas statins and aspirin can reduce CRP and decrease the risk of MI and stroke.

52. When instructing a client who has been newly diagnosed with vasospastic disorder ( Raynaud's phenomenon) about management of care, the nurse should discuss which of the following topics? ■ 1. Scheduling a sympathectomy procedure for the next visit. ■ 2. Using a beta blocker medication. ■ 3. Follow-up monitoring for development of connective tissue disease. ■ 4. Benefi t of an angioplasty to the affected extremities.

3. Clients with Raynaud's phenomenon should receive routine follow-up to monitor symptoms and to assess for the development of connective tissue or autoimmune diseases associated with Raynaud's. Beta blockers are not considered fi rstline drug therapy. A sympathectomy is considered only in advanced cases. There is no benefi t to an angioplasty, which is used for atherosclerotic vascular disease.

101. A client is admitted for a revascularization procedure for arteriosclerosis in his left iliac artery. To promote circulation in the extremities, the nurse should: ■ 1. Position the client on a fi rm mattress. ■ 2. Keep the involved extremity warm with blankets. ■ 3. Position the left leg at or below the body's horizontal plane. ■ 4. Encourage the client to raise and lower his leg four times every hour.

3. Keeping the involved extremity at or below the body's horizontal plane will facilitate tissue perfusion and prevent tissue damage. The nurse should avoid placing the affected extremity on a hard surface, such as a fi rm mattress, to avoid pressure ulcers. In addition, the involved extremity should be free from heavy overlying bed linens. The nurse should handle the involved extremity in a gentle fashion to prevent friction or pressure. Raising the leg would cause occlusion to the iliac artery, which is contrary to the goal to promote arterial circulation

71. A 70-year-old female is scheduled to undergo mitral valve replacement for severe mitral stenosis and mitral regurgitation. Although the diagnosis was made during childhood, she did not have symptoms until 4 years ago. Recently, she noticed increased symptoms, despite daily doses of digoxin and furosemide. During the initial interview with the client, the nurse would most likely learn that the client's childhood health history included: ■ 1. Chickenpox. ■ 2. Poliomyelitis. ■ 3. Rheumatic fever. ■ 4. Meningitis.

3. Most clients with mitral stenosis have a history of rheumatic fever or bacterial endocarditis. Chickenpox, poliomyelitis, and meningitis are not associated with mitral stenosis.

52. In which of the following positions should the nurse place a client with suspected heart failure? ■ 1. Semi-sitting (low Fowler's position). ■ 2. Lying on the right side (Sims' position). ■ 3. Sitting almost upright (high Fowler's position). ■ 4. Lying on the back with the head lowered (Trendelenburg's position)

3. Sitting almost upright in bed with the feet and legs resting on the mattress decreases venous return to the heart, thus reducing myocardial workload. Also, the sitting position allows maximum space for lung expansion. Low Fowler's position would be used if the client could not tolerate high Fowler's position for some reason. Lying on the right side would not be a good position for the client in heart failure. The client in heart failure would not tolerate the Trendelenburg's position.

6. A 65-year-old client is admitted to the emergency department with a fractured hip. The client has chest pain and shortness of breath. The health care provider orders nitroglycerin tablets. Which should the nurse instruct the client to do? ■ 1. Put the tablet under the tongue until it is absorbed. ■ 2. Swallow the tablet with 120 mL of water. ■ 3. Chew the tablet until it is dissolved. ■ 4. Place the tablet between his cheek and gums

3. The client is having symptoms of a myocardial infarction. The fi rst action is to prevent platelet formation and block prostaglandin synthesis. The nitroglycerin tablet will be absorbed fastest if the client chews the tablet

44. A client with angina has been taking nifedipine. The nurse should teach the client to: ■ 1. Monitor blood pressure monthly. ■ 2. Perform daily weights. ■ 3. Inspect gums daily. ■ 4. Limit intake of green leafy vegetables.

3. The client taking nifedipine should inspect the gums daily to monitor for gingival hyperplasia. This is an uncommon adverse effect but one that requires monitoring and intervention if it occurs. The client taking nifedipine might be taught to monitor blood pressure, but more often than monthly. These clients would not generally need to perform daily weights or limit intake of green leafy vegetables

43. Sublingual nitroglycerin tablets begin to work within 1 to 2 minutes. How should the nurse instruct the client to use the drug when chest pain occurs? ■ 1. Take one tablet every 2 to 5 minutes until the pain stops. ■ 2. Take one tablet and rest for 10 minutes. Call the physician if pain persists after 10 minutes. ■ 3. Take one tablet, then an additional tablet every 5 minutes for a total of three tablets. Call the physician if pain persists after three tablets. ■ 4. Take one tablet. If pain persists after 5 minutes, take two tablets. If pain still persists 5 minutes later, call the physician.

3. The correct protocol for nitroglycerin use involves immediate administration, with subsequent doses taken at 5-minute intervals as needed, for a total dose of three tablets. Sublingual nitroglycerin appears in the bloodstream within 2 to 3 minutes and is metabolized within about 10 minutes.

88. A client is admitted to the emergency department complaining of severe abdominal pain. A radiograph reveals a large abdominal aortic aneurysm. The primary goal at this time is to: ■ 1. Maintain circulation. ■ 2. Manage pain. ■ 3. Prepare the client for emergency surgery. ■ 4. Teach postoperative breathing exercises.

3. The primary goal is to prepare the client for emergency surgery. The goal would be to prevent rupture of the aneurysm and potential death. Circulation is maintained, unless the aneurysm ruptures. When the client is prepared for surgery, the nurse should place the client in a recumbent position to promote circulation, teach the client about postoperative breathing exercises, and administer pain medication if ordered.

102. Upon assessment of third degree heart block on the monitor, the nurse should fi rst: ■ 1. Call a code. ■ 2. Begin cardiopulmonary resuscitation. ■ 3. Have transcutaneous pacing ready at the bedside. ■ 4. Prepare for defi brillation.

3. Transcutaneous pads should be placed on the client with third degree heart block. For a client who is symptomatic, transcutaneous pacing is the treatment of choice. The hemodynamic stability and pulse should be assessed prior to calling a code or initiating CPR. Defi brillation is performed for ventricular fi brillation or ventricular tachycardia with no pulse.

16. The nurse is unable to palpate the client's left pedal pulses. Which of the following actions should the nurse take next? ■ 1. Auscultate the pulses with a stethoscope. ■ 2. Call the physician. ■ 3. Use a Doppler ultrasound device. ■ 4. Inspect the lower left extremity.

3. When pedal pulses are not palpable, the nurse should obtain a Doppler ultrasound device. Auscultation is not likely to be helpful if the pulse isn't palpable. Inspection of the lower extremity can be done simultaneously when palpating, but the nurse should fi rst try to locate a pulse by Doppler. Calling the physician may be necessary if there is a change in the client's condition.

A nurse is instructing a client diagnosed with coronary artery disease about care at home. The nurse determines that teaching is effective when the client states: SELECT ALL THAT APPLY. 1. "If I have chest pain, I should contact my physician immediately." 2. "I should carry my nitroglycerin in my front pants pocket so it is handy." 3. "If I have chest pain, I stop activity and place one nitroglycerin tablet under my tongue." 4. "I should always take three nitroglycerin tablets, 5 minutes apart." 5. "I plan to avoid being around people when they are smoking." 6. "I plan on walking on most days of the week for at least 30 minutes."

356 Stopping activity decreases the body's demand for oxygen. One nitroglycerin tablet, taken sublingually, dilates the coronary arteries and increases oxygen to the myocardium. If pain is unrelieved, a second tablet should be taken 5 minutes later. Passive smoke can cause vasoconstriction and decrease blood flow velocity even in healthy young adults. The American Heart Association recommends exercising for 30 minutes on most days of the week. Medical attention is required only if pain persists and then the client should call 911 rather than the physician because emergency treatment may be necessary. Nitroglycerin loses its potency if stored in warm, moist areas, making the client's pants pocket an undesirable location for storage. If pain is relieved after one tablet, another tablet is not required. The standard dose for nitroglycerin is one tablet or spray 5 minutes apart until pain is relieved, to a maximum of three tablets.

A nurse notes that a client, who experienced a myocardial infarction (MI) 3 days ago, seems unusually fatigued. Upon assessment, the nurse finds that the client is dyspneic with activity, has a heart rate (HR) of 110 beats per minute (bpm), and has generalized edema. Which action by the nurse is most appropriate? 1. Administer high-flow oxygen 2. Encourage the client to rest more 3. Continue to monitor the client's heart rhythm 4. Compare the client's admission weight with the client's current weight

4 A complication of MI is heart failure. Signs of heart failure include fatigue, dyspnea, tachycardia, edema, and weight gain. Other signs include nocturia, skin changes, behavioral changes, and chest pain. There is no indication that the client is hypoxic and in need of high-flow oxygen. To treat the dyspnea, oxygen by nasal cannula would be appropriate. The fatigue is caused by decreased cardiac output, impaired perfusion to vital organs, decreased tissue oxygenation, and anemia. Rest alone will not relieve the fatigue. Interventions are needed to improve cardiac output and tissue oxygenation. A heart rate of 110 bpm suggests tachycardia; the symptoms together imply heart failure. Further data collection is needed to confirm the findings. Continuing to monitor the client's heart rhythm, without further assessment, will delay an appropriate intervention.

A client admitted to a telemetry unit with a diagnosis of Prinzmetal's angina, has the following medications ordered. Upon interpretation of the client's electrocardiogram (ECG) rhythm, the nurse notes a prolonged PR interval of 0.32 second. Based on this information, which medication order should the nurse question administering to the client? 1. Isosorbide mononitrate (Imdur®) 20 mg oral daily upon awakening 2. Amlodipine (Norvasc®) 10 mg oral daily 3. Nitroglycerin (Nitrostat®) 0.4 mg sublingual prn for chest pain 4. Atenolol (Tenormin®) 50 mg oral daily.

4 Atenolol, a beta-blocker, blocks stimulation of beta1 (myocardial)- adrenergic receptors, causing a reduction in blood pressure and heart rate. A side effect of the medication is a prolongation of the PR interval (normal PR interval is 0.12 to 0.20 second). Continued use of the drug can result in heart block. Nitrates and calcium channel blockers (CCBs) are the mainstays of medical therapy for variant angina rather than beta blockers. Isosorbide mononitrate, a nitrate, causes vasodilatation of the large coronary arteries. Nitrates act as an exogenous source of nitric oxide, which causes vascular smooth muscle relaxation resulting in a decrease in myocardial oxygen consumption and may have a modest effect on platelet aggregation and thrombosis. Amlodipine, a CCB, relaxes coronary smooth muscle and produces coronary vasodilation, which in turn improves myocardial oxygen delivery. Nitroglycerin sublingual effectively treats episodes of angina and myocardial ischemia within minutes of administration, and the long-acting nitrate preparation (Imdur®) reduces the frequency of recurrent events.

A nurse, assessing a client hospitalized following a myocardial infarction (MI), obtains the following vital signs: blood pressure (BP) 78/38 mm Hg, heart rate (HR) 128, respiratory rate (RR) 32. For which life-threatening complication should the nurse carefully monitor the client? 1. Pulonary embolism 2. Cardiac tamponade 3. Cardiomyopathy 4. Cardiogenic shock

4 The symptoms are indicative of cardiogenic shock (decreased cardiac output leading to inadequate tissue perfusion and initiation of the shock syndrome). Pulmonary embolism, cardiac tamponade, and cardiomyopathy are causes of cardiogenic shock. The cause of this client's cardiogenic shock is a myocardial infarction.

39. The client is scheduled for a percutaneous transluminal coronary angioplasty (PTCA) to treat angina. Priority goals for the client immediately after PTCA should include: ■ 1. Minimizing dyspnea. ■ 2. Maintaining adequate blood pressure control. ■ 3. Decreasing myocardial contractility. ■ 4. Preventing fl uid volume defi cit.

4. Because the contrast medium used in PTCA acts as an osmotic diuretic, the client may experience diuresis with resultant fl uid volume defi cit after the procedure. Additionally, potassium levels must be closely monitored because the client may develop hypokalemia due to the diuresis. Dyspnea would not be anticipated after this procedure. Maintaining adequate blood pressure control should not be a problem after the procedure. Increased myocardial contractility would be a goal, not decreased contractility

4. Peripheral blood fl ow is dependent on which of the following variables? ■ 1. Blood viscosity and diameter of vessels. ■ 2. Diameter and resistance of vessels. ■ 3. Force of contraction of the heart and resistance of vessels. ■ 4. Pressure differences in the arterial and venous systems and resistance.

4. Blood fl ows in a unidirectional manner, and the blood fl ow involves the differences in pressure between the arterial and venous systems. The two variables infl uencing blood fl ow within this closed system are the pressure differences and the resistance to blood fl ow throughout the system. The greater the resistance, the greater the driving force needed, which results in an increase in the force of the contraction of the heart. Blood viscosity is important, and diameter infl uences resistance, but fl ow is dependent on pressure differences and resistance. The force of the contraction of the heart and resistance of vessels infl uence fl ow, but it is the pressure differences that control blood fl ow

75. A client is admitted to the unit with a diagnosis of thrombophlebitis and deep vein thrombosis of the right leg. A loading dose of heparin has been given in the emergency room, and I.V. heparin will be continued for the next several days. The nurse should develop a plan of care for this client that will involve: ■ 1. Administering aspirin as ordered. ■ 2. Encouraging green leafy vegetables in the diet. ■ 3. Monitoring the client's prothrombin time (PT). ■ 4. Monitoring the client's activated partial thromboplastin time (aPTT) and International Normalized Ratio (INR).

4. Heparin dosage is usually determined by the physician based on the client's aPTT and INR laboratory values. Therefore, the nurse monitors these values to prevent complications. Administering aspirin when the client is on heparin is contraindicated. Green leafy vegetables are high in vitamin K and therefore are not recommended for clients receiving heparin. Monitoring of the client's PT is done when the client is receiving warfarin sodium (Coumadin

75. In preparing the client and the family for a postoperative stay in the intensive care unit (ICU) after open heart surgery, the nurse should explain that: ■ 1. The client will remain in the ICU for 5 days. ■ 2. The client will sleep most of the time while in the ICU. ■ 3. Noise and activity within the ICU are minimal. ■ 4. The client will receive medication to relieve pain.

4. Management of postoperative pain is a priority for the client after surgery, including valve replacement surgery, according to the Agency for Health Care Policy and Research. The client and family should be informed that pain will be assessed by the nurse and medications will be given to relieve the pain. The client will stay in the ICU as long as monitoring and intensive care are needed. Sensory deprivation and overload, high noise levels, and disrupted sleep and rest patterns are some environmental factors that affect recovery from valve replaceent surgery.

87. A client had a repair of a thoracoabdominal aneurysm 2 days ago. Which of the following fi ndings should the nurse consider unexpected and report to the physician immediately? The client has: ■ 1. Abdominal pain at 5 on a scale of 0 to 10 for the last 2 days. ■ 2. Heart rate of 100 beats per minute after ambulating 200 feet. ■ 3. Urine output of 2,000 mL in 24 hours. ■ 4. Weakness and numbness in the lower extremities.

4. One of the complications of a thoracoabdominal aneurysm repair is spinal cord injury. Therefore, it is important for the nurse to assess for signs and symptoms of neurologic changes at and below the site where the aneurysm was repaired. The client is expected to have moderate pain following surgery An elevated heart rate is expected after physical exertion. It is important to monitor urine output following aneurysm surgery, but a urine output of 2,000 mL in 24 hours is adequate following surgery.

29. A client who has been diagnosed with peripheral vascular disease (PVD) is being discharged. The client needs further instruction if she says she will: ■ 1. Avoid heating pads. ■ 2. Not cross her legs. ■ 3. Wear leather shoes. ■ 4. Use iodine on an injured site.

4. The client should avoid using iodine or over-the-counter medications. Iodine is a highly toxic solution. An individual who has known PVD should be seen by a physician for treatment to avoid infection. The client with PVD should avoid heating pads and crossing the legs, and should wear leather shoes. A heating pad can cause injury, which, because of the decreased blood supply, can be diffi cult to heal. Crossing the legs can further impede blood fl ow. Leather shoes provide better protection.

8. The nurse has just admitted a 35-year-old female client who has a serum vitamin B12 concentration of 800 pg/mL. Which of the following laboratory fi ndings should cue the nurse to focus the client history assessment on specifi c drug or alcohol use? ■ 1. Total bilirubin, 0.3 mg/dL. ■ 2. Serum creatinine, 0.5 mg/dL. ■ 3. Hemoglobin, 16 g/dL. ■ 4. Folate, 1.5 ng/mL.

4. The normal range of folic acid is 1.8 to 9 ng/mL, and the normal range of vitamin B12 (cyanocobalamin) is 200 to 900 pg/mL. A low folic acid level in the presence of a normal vitamin B12 level is indicative of a primary folic acid defi ciency anemia. Factors that affect the absorption of folic acid are drugs such as methotrexate, oral contraceptives, antiseizure drugs, and alcohol. The total bilirubin, serum creatinine, and hemoglobin values are within normal limits.

56. The nurse teaches a client with heart failure to take oral furosemide in the morning. The primary reason for this is to help: ■ 1. Prevent electrolyte imbalances. ■ 2. Retard rapid drug absorption. ■ 3. Excrete excessive fl uids accumulated during the night. ■ 4. Prevent sleep disturbances during the night.

4. When diuretics are given early in the day, the client will void frequently during the daytime hours and will not need to void frequently during the night. Therefore, the client's sleep will not be disturbed. Taking furosemide in the morning has no effect on preventing electrolyte imbalances or retarding rapid drug absorption. The client should not accumulate excessive fl uids throughout the night.

83. A client has an emergency embolectomy for an embolus in the femoral artery. After the client returns from the recovery room, in what order, from fi rst to last, should the nurse provide care? 1. Administer pain medication 2. Draw blood for laboratory studies. 3. Regulate the I.V. infusion. 4. Monitor the pulses. 5. Inspect the dressing.

45312 The nurse should fi rst monitor the popliteal and pedal pulses in the affected extremity after arterial embolectomy. Monitoring peripheral pulses below the site of occlusion checks the arterial circulation in the involved extremity. The nurse should next inspect the dressing to be sure that the client is not bleeding at the surgical site. The nurse should next regulate the I.V. infusion to prevent fl uid overload. Then the nurse should assess pain and administer pain medications as ordered. Last, the nurse can obtain blood for laboratory studies.


Related study sets

Money and Banking Chapter 18 and 20

View Set

Intro to Public Speak. - Mid-Term Questions

View Set

Chapter 12-Postpartum physiological assessments and nursing care

View Set